Решение уравнение с двумя переменными: определение, решение, свойства и примеры

3+y = 7$

Решением уравнения с двумя переменными называется упорядоченная пара значений переменных (x,y), обращающая это уравнение в тождество.

О тождествах – см. §3 данного справочника

Например: для уравнения 2x+5y=6 решениями являются пары

x = -2, y = 2; x = -1,y = 1,6; x = -3,y = 2,4 и т.д.

Уравнение имеет бесконечное множество решений.

Содержание

Свойства уравнения с двумя переменными

Уравнения с двумя переменными, имеющие одни и те же решения, называют равносильными. Уравнения с двумя переменными, не имеющие решений, также считают равносильными.

Уравнения с двумя переменными имеют такие же свойства, как и уравнения с одной переменной:

  • если в уравнении перенести слагаемое из одной части в другую и изменить его знак, получится уравнение, равносильное данному;
  • если обе части уравнения умножить или разделить на одно и то же, отличное от нуля число, то получится уравнение, равносильное данному.

Например: $2x+5y = 6 ⟺5y = -2x+6 \iff y = -0,4x+1,2$

Примеры

Пример 1. Из данного линейного уравнения выразите y через x и x через y:

Алгоритм: рассмотрим 3x+4y=10

1) оставим слагаемое с выражаемой переменной с одной стороны, остальные слагаемые перенесем в другую сторону: 4y=-3x+10

2) разделим полученное уравнение слева и справа на коэффициент при выражаемой переменной: y=-0,75x+2,5 — искомое выражение y(x).

Аналогично для x(y): $3x+4y = 10 \iff 3x = -4y+10 \iff x = -1 \frac{1}{3} y+3 \frac{1}{3}$

Линейное уравнение

$x = \frac{2}{3} y+3 \frac{2}{3}$

$ y = — \frac{x}{7}+1 \frac{1}{7}$

Пример 2. Составьте линейное уравнение с двумя переменными, решением которого является пара чисел:

Алгоритм: рассмотрим (1;5)

1) составим любой двучлен вида ax+by, например 2x+3y

2) подставим данные x = 1, y = 5 в двучлен и запишем результат 2x+3y = 17 — это искомое уравнение.

Пример 3. Составьте уравнение с двумя переменными, решениями которого являются две пары чисел:

а) (1;5) и (2;4)

Искомое уравнение имеет вид ax+by=c. Подставим обе пары:

$$ {\left\{ \begin{array}{c} a+5b = c \\ 2a+4b = c \end{array} \right.} \Rightarrow a+5b = 2a+4b \Rightarrow a = b $$

Пусть a = b = 1. Тогда x+y = 1+5 = 2+4 = 6

x+y = 6 — искомое уравнение.

б) (0;2) и (2;5)

Искомое уравнение имеет вид ax+by = c. Подставим обе пары:

$$ {\left\{ \begin{array}{c} 0+2b = c \\ 2a+5b = c \end{array} \right.} \Rightarrow 2b = 2a+5b \Rightarrow a = -1,5b $$

Пусть b = -2. Тогда a = 3 и уравнение:

$3x-2y = 3\cdot0-2\cdot2 = 3\cdot2-2\cdot5 = -4$

3x-2y = -4 — искомое уравнение.

Пример 4. Найдите двузначное число, которое в два раза больше суммы своих цифр.

Пусть a-цифра десятков (a = 1,2,…,9), b- цифра единиц (b = 0,1,…,9).

По условию: 10a+b = 2(a+b)

$$10a+b = 2a+2b \Rightarrow 8a = b$$

Единственное возможное решение: a = 1, b = 8

Ответ:18

Пример 5. Найдите двузначное число, которое при умножении на сумму своих цифр даёт 370.

Пусть a-цифра десятков (a = 1,2,…,9), b- цифра единиц (b = 0,1,…,9).

По условию: (10a+b)(a+b) = 370

Разложим 370 на простые множители: $370 = 2\cdot5\cdot37$

Возможные значения для суммы a+b = {2;5;10}

Рассмотрим a+b = 2. Тогда 10a+b = $\frac{370}{a+b} = \frac{370}{2} = 185 — не \quad двузначное \quad число \Rightarrow$

$a+b \neq 2$

Рассмотрим a+b = 5. Тогда 10a+b = $\frac{370}{5} = 74 \Rightarrow a = 7, b = 4, a+b \neq 5$.

Рассмотрим a+b = 10. Тогда 10a+b = $\frac{370}{10} = 37 \Rightarrow a = 3, b = 7, a+b = 10$.

Значит, искомое число 37.

Ответ: 37

Подготовка школьников к ЕГЭ и ОГЭ (Справочник по математике — Алгебра

Линейные уравнения (уравнения первой степени) с двумя неизвестными

      Определение 1. Линейным уравнением (уравнением первой степени) с двумя неизвестными   x   и   y   называют уравнение, имеющее вид

где   a ,  b ,  c   – заданные числа.

      Определение 2. Решением уравнения (1) называют пару чисел   (y) ,   для которых формула (1) является верным равенством.

      Пример 1. Найти решение уравнения

      Решение. Выразим из равенства (2) переменную   y   через переменную   x :

(3)

      Из формулы (3) следует, что решениями уравнения (2) служат все пары чисел вида

где   x   – любое число.

      Замечание. Как видно из решения примера 1, уравнение (2) имеет бесконечно много решений. Однако важно отметить, что не любая пара чисел   (y)   является решением этого уравнения. Для того, чтобы получить какое-нибудь решение уравнения (2), число   x   можно взять любым, а число   y   после этого вычислить по формуле (3).

Системы из двух линейных уравнений с двумя неизвестными

      Определение 3. Системой из двух линейных уравнений с двумя неизвестными   x   и   y   называют систему уравнений, имеющую вид

(4)

где   a1 ,  b1 ,  c1 ,  a2 ,  b2 ,  c2   – заданные числа.

      Определение 4. В системе уравнений (4) числа   a1 ,  b1 a2 ,  b2   называют коэффициентами при неизвестных, а числа   c1 ,  c2  – свободными членами.

      Определение 5. Решением системы уравнений (4) называют пару чисел   (y) ,   являющуюся решением как одного, так и другого уравнения системы (4).

      Определение 6. Две системы уравнений называют равносильными (эквивалентными), если все решения первой системы уравнений являются решениями второй системы, и все решения второй системы являются решениями первой системы.

      Равносильность систем уравнений обозначают, используя символ «»

      Системы линейных уравнений решают с помощью метода последовательного исключения неизвестных, который мы проиллюстрируем на примерах.

      Пример 2 . Решить систему уравнений

(5)

      Решение. Для того, чтобы решить систему (5) исключим из второго уравнения системы неизвестное   х.

      С этой целью сначала преобразуем систему (5) к виду, в котором коэффициенты при неизвестном   x   в первом и втором уравнениях системы станут одинаковыми.

      Если первое уравнение системы (5) умножить на коэффициент, стоящий при   x   во втором уравнении (число   7 ), а второе уравнение умножить на коэффициент, стоящий при   x   в первом уравнении (число   2 ), то система (5) примет вид

 

(6)

      Теперь совершим над системой (6) следующие преобразования:

  • первое уравнение системы оставим без изменений;
  • из второго уравнения вычтем первое уравнение и заменим второе уравнение системы на полученную разность.

      В результате система (6) преобразуется в равносильную ей систему

      Из второго уравнения находим   y = 3 ,   и, подставив это значение в первое уравнение, получаем

      Ответ.   (–2 ; 3) .

      Пример 3. Найти все значения параметра   p ,   при которых система уравнений

(7)

      а) имеет единственное решение;

      б) имеет бесконечно много решений;

      в) не имеет решений.

      Решение. Выражая   x   через   y   из второго уравнения системы (7) и подставляя полученное выражение вместо   x   в первое уравнение системы (7), получим

      Следовательно, система (7) равносильна системе

(8)

      Исследуем решения системы (8) в зависимости от значений параметра   p .   Для этого сначала рассмотрим первое уравнение системы (8):

y (2 – p) (2 + p) = 2 + p(9)

      Если   ,   то уравнение (9) имеет единственное решение

      Следовательно, система (8) равносильна системе

      Таким образом, в случае, когда   ,   система (7) имеет единственное решение

      Если   p = – 2 ,   то уравнение (9) принимает вид

,

и его решением является любое число . Поэтому решением системы (7) служит бесконечное множество всех пар чисел

,

где   y   – любое число.

      Если   p = 2 ,   то уравнение (9) принимает вид

и решений не имеет, откуда вытекает, что и система (7) решений не имеет.

Системы из трех линейных уравнений с тремя неизвестными

      Определение 7. Системой из трех линейных уравнений с тремя неизвестными   x ,   y     и   z   называют систему уравнений, имеющую вид

(10)

где   a1 ,  b1 ,  c1 ,  d1 ,  a2 ,  b2 ,  c2 ,  d2 ,  a3 ,  b3 ,  c3 ,  d3   – заданные числа.

      Определение 8. В системе уравнений (10) числа   a1 ,  b1 ,  c1 ,  a2 ,  b2 ,  c2 ,  a3 ,  b3 ,  c3   называют коэффициентами при неизвестных, а числа   d1 ,  d2 ,  d3   – свободными членами.

      Определение 9. Решением системы уравнений (10) называют тройку чисел   (y ; z) ,   при подстановке которых в каждое из трех уравнений системы (10) получается верное равенство.

      Пример 4 . Решить систему уравнений

(11)

      Решение. Будем решать систему (11) при помощи метода последовательного исключения неизвестных.

      Для этого сначала исключим из второго и третьего уравнений системы неизвестное   y ,  совершив над системой (11) следующие преобразования:

  • первое уравнение системы оставим без изменений;
  • ко второму уравнению прибавим первое уравнение и заменим второе уравнение системы на полученную сумму;
  • из третьего уравнения вычтем первое уравнение и заменим третье уравнение системы на полученную разность.

      В результате система (11) преобразуется в равносильную ей систему

(12)

      Теперь исключим из третьего уравнения системы неизвестное   x ,  совершив над системой (12) следующие преобразования:

  • первое и второе уравнения системы оставим без изменений;
  • из третьего уравнения вычтем второе уравнение и заменим третье уравнение системы на полученную разность.

      В результате система (12) преобразуется в равносильную ей систему

(13)

      Из системы (13) последовательно находим

z = – 2 ;   x = 1 ;   y = 2 .

      Ответ.   (1 ; 2 ; –2) .

      Пример 5. Решить систему уравнений

(14)

      Решение. Заметим, что из данной системы можно получить удобное следствие, сложив все три уравнения системы:

      Если числа   (y ; z)   являются решением системы (14), то они должны удовлетворять и уравнению (15). Однако в таком случае числа   (y ; z)   должны также быть решением системы, которая получается, если из каждого уравнения системы (14) вычесть уравнение (15):

      Поскольку мы использовали следствие из системы (14), не задумываясь о том, являются ли сделанные преобразования системы (14) равносильными, то полученный результат нужно проверить. Подставив тройку чисел   (3 ; 0 ; –1)   в исходную систему (14), убеждаемся, что числа   (3 ; 0 ; –1)   действительно являются ее решением.

      Ответ:   (3 ; 0 ; –1) .

      Замечание. Рекомендуем посетителю нашего сайта, интересующемуся методами решения систем уравнений, ознакомиться также c разделом справочника «Системы с нелинейными уравнениями» и нашим учебным пособием «Системы уравнений».

      На нашем сайте можно также ознакомиться нашими учебными материалами для подготовки к ЕГЭ и ОГЭ по математике.

Решение задач. Уравнение с двумя неизвестными

Здравствуйте уважаемые обучающиеся и их родители. 

На прошлых занятиях мы говорили по теме «Уравнения с двумя неизвестными». Обсудили, что называют уравнением с двумя неизвестными, как решать такие уравнения, что является решением такого уравнения. 

Обо всем по теме мы говорили на странице: 

Сегодня мы потренируемся в решении задач на решение уравнений с двумя неизвестными.

Давайте посмотрим ролик, где рассказывают о том, как решать уравнение с двумя неизвестными:

Видео YouTube


Видео YouTube

Давайте познакомимся с основными положениями темы, которые нужно знать. Я их отразил в виде схем для наглядности.

Помним о том, как задаются координаты. Координаты записываются в круглых скобках, отражают положение точки в плоскости по оси х и y. Ось абсцисс — х, ось ординат — у.

Построим некоторые точки….

По новой теме:Начало.

Переходим с разбору заданий: В тетрадку оформим:

Разбор модели задания:

Решение задачи № 2. Разбор в тетрадь. Ощутите алгоритм, последовательность решения.  

Запишите алгоритм.

Разбор заданий:

Задача № 1. Выразить в  уравнении одну переменную через другую.

Задача № 2. Построить уравнения с двумя неизвестными. Построено три уравнения. Графики — все линейные функции.

Для построения достаточно 2 точек, две пары чисел (x, y), которые являются решениями уравнений ниже.

Задача № 2. Алгоритм в действии. Построение одного из множества графиков уравнения с двумя неизвестными.

Выразили переменную y через х, после чего уравнение поделили все на 2. Упростили его. Получилось у=-2х+4. После чего взяли произвольные точки,которые являются решением этого уравнения и построили прямую — линейную функцию.

Домашняя работа: Работа состоит из 10 заданий. Половина из них решается в одну строчку, поэтому не думайте, что они сложные. Они основные и базовые по этой теме. Их нужно уметь делать. Все, что нужно для решения, указано в статье. Там все. Для того, чтобы разобраться, статью нужно прочесть, видео посмотреть, иначе нельзя.

Урок 42. линейные уравнения и неравенства с двумя переменными — Алгебра и начала математического анализа — 11 класс

Алгебра и начала математического анализа, 11 класс

Урок №42. Линейные уравнения и неравенства с двумя переменными

Перечень вопросов, рассматриваемых в теме:

  • Решение уравнений, неравенств, систем уравнений и систем неравенств с двумя переменными;
  • Изображение в координатной плоскости множества решений уравнений, неравенств, систем уравнений, систем неравенств;
  • Нахождение площади получившейся фигуры.

Глоссарий по теме

Уравнение вида     ax + by + c = 0 называется линейным уравнением с двумя переменными, где   a, b и c   —   некоторые числа (a ≠ 0 ,   b ≠0), а, х и у   —   переменные. 

Основная литература:

Колягин Ю.М., Ткачева М.В., Федорова Н.Е. и др., под ред. Жижченко А. Б. Алгебра и начала математического анализа (базовый и профильный уровни) 11 кл. – М.: Просвещение, 2014.

Дополнительная литература:

Макарычев Ю.Н., Миндюк Н.Г., Нешков К.И. Учебник: Алгебра 9 кл с углубленным изучением математики Мнемозина, 2014.

Открытые электронные ресурсы:

Решу ЕГЭ образовательный портал для подготовки к экзаменам https://ege.sdamgia.ru/.

Открытый банк заданий ЕГЭ ФИПИ, Элементы комбинаторики, статистики и теории вероятностей, базовый уровень. Элементы комбинаторики, статистики и теории вероятностей. Базовый уровень. http://ege.fipi.ru/.

Теоретический материал для самостоятельного изучения

Историческая справка

Уравнения, а также системы уравнений имеют давнюю историю. Нам известно, что уже в Древнем Вавилоне и Индии повседневные задачи, связанные с земляными работами или планированием военных расходов, а также астрономическими наблюдениями решались с помощью уравнений и их систем.

В то время еще не существовало привычного нам формального языка математики. Вавилоняне, также, как и индусы не использовали в своих трактатах привычные нам «икс» и «игрек». Не обозначали степень надстрочными индексами. И т.д. Их уравнения записаны в виде текстовых задач. Также, как и решения, не похожи на современные, а скорее напоминают цепочку логических рассуждений.

Вместе с тем, если перевести в привычный нам вид те уравнения, которые умели решать в Древнем Вавилоне, то мы увидим: . И в древнем индийском манускрипте «Ариабхаттиам», датируемом 499 годом нашей эры, также встречаются задачи, решаемые с помощью квадратных уравнений. Индийские мудрецы (слово ученый тоже еще не существовало) уже не ограничивались решением конкретных житейских задач, но и работали над решением квадратного уравнения в общем виде.

Привычный нам вид уравнения обретают только в конце шестнадцатого века, благодаря трудам Франсу Виета (1540 – 1603 гг.). Именно он, помимо прочих своих научных достижений обладает и неофициальным титулом «создатель алгебры». Поскольку разработал и активно внедрял символический язык алгебры – те самые, привычные нам «иксы и игреки».

Актуализация знаний

1.Найдите уравнения, которые являются линейными.

4х + 5у = 10; ; у = 7х +4

Ответ: 4х + 5у = 10; у = 7х +4

Сегодня на уроке мы вспомним что такое линейные уравнения и неравенства с двумя переменными; системы линейный уравнений и неравенств, а также научимся изображать множество на плоскости, задаваемое линейным уравнением и неравенством.

  1. Линейные уравнения с двумя переменными.

Уравнение вида ах + by +с =0, где а,b,с – некоторые числа, называется линейным уравнением с двумя переменными х и у.

Решением уравнения ах + by +с =0, где а,b,с – некоторые числа, называется пара значений обращающая уравнение в верное числовое равенство.

Если одновременно а и b, то уравнение ах + by +с =0 является уравнением некоторой прямой. Для построения прямой достаточно найти две точки этой прямой.

Пример

Построить график уравнения 2х+у =1

у = -2х + 1

Если х=0, то у=1;

Если х=2, то у=-3.

На координатной плоскости отметим точки с координатами (0;1) и (2;-3). Через две точки на плоскости проведем прямую. Полученная прямая является геометрической моделью уравнения 2х+у =1.

  1. Линейные неравенства с двумя переменными.

Линейным неравенством с двумя переменными называется неравенство вида ах + bу + с < 0 или ах + bу + с > 0, где х и у – переменные, а, b, c – некоторые числа.

Решением неравенства с двумя переменными называется пара значений переменных, обращающая его в верное равенство.

Является ли пара (2;1) решением неравенства 5х + 2у > 4 . Является, тк при подстановке в него вместо х числа 2, а вместо у числа 1 получается верное равенство 10 + 2 > 4.

Если каждое решение неравенства с двумя переменными изобразить точкой в координатной плоскости, то получится график этого неравенства. Он является некоторой фигурой.

Пример

Найти множество точек координатной плоскости, удовлетворяющих неравенству 3х – 2у +6 > 0.

  1. Уравнение 3х – 2у +6 = 0 является уравнением прямой, проходящей через точки(- 2; 0) и (0; 3).
  2. Пусть точка М11,у1) лежит в заштрихованной полуплоскости (ниже прямой 3х – 2у +6 = 0, аМ21,у2)лежит на прямой 3х – 2у +6 = 0. Тогда 2у2 – 3х1 – 6 = 0, а 2у1 – 3х1 – 6 < 0, т.к. у1< у2

Изобразим множество точек координатной плоскости, удовлетворяющих неравенству 3х – 2у +6 > 0 штриховкой (рис. 1)

Рисунок 1 – решение неравенства 3х – 2у +6 > 0

Если в линейном неравенстве с двумя переменными знак неравенства заменить знаком равенства, то получится линейное уравнение ах + by +с =0, графиком которого является прямая при условии, что и . Прямая разбивает плоскость на две полуплоскости. Одна из них является графиком неравенства ах + bу + с < 0, а другая – графиком неравенства ах + bу + с > 0

Чтобы решить неравенство ах + bу + c < 0 или aх + bу + c > 0, достаточно взять какую-нибудь точку М11; у1), не лежащую на прямой aх + bу + c = 0, и определить знак числа aх1 + bу1 + c.

Пример

Изобразите в координатной плоскости множества решений неравенства 2х + 3у < 6

Начертим график уравнения 2х + 3у = 6.

Пара (0;0) является решением неравенства 2х + 3у < 6, и принадлежит нижней полуплоскости, значит графиком неравенства 2х + 3у < 6 является нижняя полуплоскость (рис. 2).

Рисунок 2 – решение неравенства 2х + 3у < 6

  1. Система линейных уравнений с двумя переменными.

Система вида , где а,b,с,d,e,f – некоторые числа, называется линейной системой с двумя переменными х и у.

Пара значений переменных, обращающая каждое уравнение системы уравнений с двумя переменными в верное равенство называют решением системы.

Решить систему – значит найти множество ее решений.

Пример

Решите систему:

Каждое решение уравнения с двумя переменными представляет координаты некоторой его точки его графика. Каждое решение системы есть координаты общих точек графиков уравнений системы. Построим графики этих уравнений и найдем координаты точки пересечения (рис.3). 

Рисунок 3 – решение системы

Система имеет единственное решение: x = 4 ,   y = 4 .

  1. Система линейных неравенств с двумя переменными.

Системой линейных неравенств с двумя переменными называется такая система неравенств, которая в своем составе имеет два и более линейных неравенств с двумя переменными.
Рассмотрим систему линейных неравенств с двумя переменными на примере:

  1. Построим прямые х – у = 2 и х + 3у = 6
  2. Пара (4;1) является решением как первого, так и второго неравенства, те является общим решением неравенств системы. Такую пару чисел называют решением системы неравенств с двумя переменными. Множество общих решений неравенств есть множество решений системы (пересечение множеств решений неравенств, составляющих систему).

Множество решение системы изображается двойной штриховкой. (плоской угол) (рис. 4).

Рисунок 4 – решение системы

Примеры и разбор решения заданий тренировочного модуля

Пример 1

Изобразите в координатной плоскости множества решений неравенства 3х – 2у + 6 0.

  1. Начертим график уравнения 3х – 2у + 6 = 0
  2. Отметим в какой-нибудь полуплоскости, например, точку (1;2).

Пара (1;2) не является решением неравенства и принадлежит нижней полуплоскости, значит графиком неравенства является верхняя полуплоскость вместе с прямой 3х – 2у + 6 = 0. 9 (рис. 5)

Рисунок 5 – решение неравенства

Пример 2

Изобразим на координатной плоскости множество решений системы

Построим прямые х + у = 3 и 4х – 5у = 20.

Множество решений первого неравенства показано горизонтальной штриховкой, а множество решений второго неравенства – вертикальной штриховкой. Двойная штриховка – множество решений системы. Система задает плоский угол (рис. 6)

Рисунок 6 – решение системы

Если к системе добавить еще одно неравенство

, то получится система трех неравенств с двумя переменными

Этой системой задается треугольник (рис. 7)

Рисунок 7 – решение системы

Точка О принадлежит , левая часть неравенства положительна, и поэтому множество его решений – объединение множеств .

Линейное уравнение с двумя переменными: решение и свойства

 

Линейное уравнение с двумя переменными — любое уравнение, которое имеет следующий вид: a*x + b*y =с. Здесь x и y есть две переменные, a,b,c – некоторые числа.

Ниже представлены несколько примеров линейных уравнений.

1. 10*x + 25*y = 150;

2. x-y=5;

3. -7*x +y = 5;

Как и уравнения с одним неизвестным, линейное уравнение с двумя переменными (неизвестными) тоже имеет решение. Например, линейное уравнение x-y=5, при x=8 и y=3 превращается в верное тождество 8-3=5. В таком случае говорят, что пара чисел x=8 и y=3 является решением линейного уравнения x-y=5. Еще можно говорить, что пара чисел x=8 и y=3 удовлетворяет линейному уравнению x-y=5.

Решение линейного уравнения

Таким образом, решением линейного уравнения a*x + b*y = с , называется, любая пара чисел (x,y) которая удовлетворяет этому уравнению, то есть обращает уравнение с переменными x и y в верное числовое равенство.  Обратите внимание, как здесь записана пара чисел х и у. Такая запись короче и удобнее. Следует только помнить, что на первом месте в такой записи стоит значение переменной х, а на втором – значение переменной у.

Обратите внимание на то, что числа x=11 и y=8, x=205 и y=200 x= 4.5 и y= -0.5 тоже удовлетворяют линейному уравнению х-у=5, а следовательно являются решениями этого линейного уравнения. 

Решение линейного уравнения с двумя неизвестными не является единственным. Каждое линейное уравнение с двумя неизвестными имеет бесконечно много различных решений. То есть существует бесконечно много различных двух чисел х и у, которые обращают линейное уравнение в верное тождество.

Если несколько уравнений с двумя переменными имеют одинаковые решения, то такие уравнения называются равносильными уравнениями. Следует отметить, что если уравнения с двумя неизвестными не имеют решений, то их тоже считают равносильными.

Основные свойства линейных уравнений с двумя неизвестными

1. Любое из слагаемых в уравнении можно перенести из одной части в другую, при этом необходимо изменить его знак на противоположный. Полученное уравнение будет равносильно исходному.

2. Обе части уравнения можно разделить на любое число, которое не равно нулю. В результате получим уравнение равносильное исходному.

Нужна помощь в учебе?



Предыдущая тема: Применение различных способов для разложения на множители
Следующая тема:&nbsp&nbsp&nbspГрафик линейного уравнения с двумя переменными: алгоритм построения

Линейное уравнение с двумя переменными

Вопросы занятия:

·  повторить что такое линейное уравнение с одной переменной и сколько решений может иметь такое уравнение;

·  ввести понятия «линейное уравнение с двумя переменными», «решение уравнения с двумя переменными», «равносильные уравнения».

Материал урока

Ранее мы с вами рассматривали линейное уравнение с одной переменной.

Вспомним, что:

Сегодня на уроке мы познакомимся с линейным уравнением, но уже с двумя неизвестными.

Давайте рассмотрим ситуацию

Полученное равенство содержит две переменные. А поэтому такие равенства называют уравнениями с двумя переменными (или с двумя неизвестными).

Посмотрите на примеры уравнений с двумя переменными

Сформулируем определение:

Определение.

Линейным уравнением с двумя переменными называется уравнение вида:

Вернёмся к задаче

То есть пара значений переменных (x = 60, y = 110) является решением этого уравнения. Отметим, что эти корни были найдены методом подбора, причём это не единственная пара чисел, удовлетворяющих нашему уравнению.

Определение.

Решением уравнения с двумя переменными называется пара значений переменных, которая обращает это уравнение в верное равенство.

Вспомним, что при изучении уравнений с одной переменной, мы говорили о равносильных уравнениях, то есть уравнениях, которые имеют одни и те же корни.

Аналогично можем сказать, что уравнения с двумя переменными, имеющие одни и те же решения, называются равносильными.

Причем уравнения с двумя переменными, не имеющие решений, также являются равносильными.

Равносильные уравнения обладают следующими свойствами:

Свойство 1.

Если в уравнении перенести слагаемое из одной части в другую, изменив его знак, то получится уравнения, равносильное данному;

Свойство 2.

Если обе части уравнения умножить (или разделить) на одно и то же отличное от нуля число, то получится уравнение, равносильное данному.

Снова вернёмся к нашему уравнению

Но здесь важно знать, значение какой из переменных стоит на первом месте, а какой – на втором. Так в нашем случае сначала записано значение переменной x, а затем переменной y.

При этом пара чисел (150; — 25) являясь решением уравнения, не удовлетворяет условию задачи, так как скорость автомобиля не может быть отрицательной.

И давайте рассмотрим ещё одну задачу.

Пример.

Решение уравнений в целых числах, то есть когда надо найти только целые значения переменных, подробно рассматривал древнегреческий математик Диофант.

Поэтому уравнения с несколькими переменными, которые надо решить в целых числах, называют диофантовыми уравнениями. То есть уравнение, составленное в предыдущей задаче, является диофантовым, так как для него мы отыскивали только натуральные решения.

И давайте рассмотрим примеры.

Пример.

И ещё пример.

Пример.

Итоги урока

Итак, на этом уроке мы рассмотрели линейное уравнение с двумя переменными и один из способов решения таких уравнений.

 

Решение уравнений с двумя неизвестными онлайн калькулятор

Применение уравнений широко распространено в нашей жизни. Они используются во многих расчетах, строительстве сооружений и даже спорте. Уравнения человек использовал еще в древности и с тех пор их применение только возрастает. Наверняка многие знают, что уравнение представляет собой некое тождество с неизвестной, которую необходимо определить, чтобы решить уравнение и получить равные значения левой и правой частей. Чтобы решить данного рода уравнения необходимо перенести в левую сторону все известные значения, а в правую все неизвестные. Решить данные уравнения можно с помощью 3 методов:

1) подстановки;

2) сложения;

3) построения графиков.

Выбор метода зависит от целевого уравнения. Решить онлайн уравнение с двумя неизвестными можно на многих сайтах, однако слепо доверять полученному результату не стоит.

Так же читайте нашу статью «Решить уравнение с 3 неизвестными онлайн»

Ниже приведен пример решения уравнения с 2 неизвестными методом сложения.

\[2x — 5y = 61\]

\[-9x + 5y = -40\]

Первое, с чего стоит начать решение — сложить каждое слагаемое с учетом их знаков:

\[2x + (-9x) = -7x\]

\[-5y + 5y = 0\]

\[61 + (-40) = 21\]

В большинстве случаев, одна из сумм, включающая в себя неизвестную будет содержать величину, равную нулю. На следующем этапе решения уравнения нам необходимо составить уравнение из полученных данных:

\[-7x + 0 = 21\]

Найти неизвестное:

\[-7x = 21, x = 21 \div (-7) = -3\]

Вставить полученное значение в любое из исходных уравнений и получить 2 неизвестное с помощью решения уравнения линейного типа:

\[2x — 5y = 61\]

\[2(-3) — 5y = 61\]

\[-6 — 5y = 61\]

\[-5y = 61 + 6\]

\[-5y = 67\]

\[y = -13,4\]

Конечный результат:

\[x = -3, y = -13,4\]

Где можно решить уравнение с 2 неизвестными онлайн?

Решить уравнение с двумя неизвестными онлайн решателем можно на сайте https://pocketteacher. ru. Бесплатный онлайн решатель позволит решить уравнение онлайн любой сложности за считанные секунды. Все, что вам необходимо сделать — это просто ввести свои данные в решателе. Так же вы можете посмотреть видео инструкцию и узнать, как решить уравнение на нашем сайте. А если у вас остались вопросы, то вы можете задать их в нашей групе Вконтакте http://vk.com/pocketteacher. Вступайте в нашу группу, мы всегда рады помочь вам.

Решение систем уравнений с двумя переменными (Алгебра 2, Как решить систему линейных уравнений) — Mathplanet

Система линейного уравнения состоит из двух или более уравнений, одно из которых ищет общее решение этих уравнений. В системе линейных уравнений каждому уравнению соответствует прямая линия, и каждый ищет точку, где две линии пересекаются.


Пример

Решите следующую систему линейных уравнений:

$$ \ left \ {\ begin {matrix} y = 2x + 4 \\ y = 3x + 2 \\ \ end {matrix} \ right.

$

Поскольку мы ищем точку пересечения, мы можем построить график уравнений:

Здесь мы видим, что линии пересекаются друг с другом в точке x = 2, y = 8. Это наше решение, и мы можем называть его графическим решением задачи.

Но как найти решение, если линии никогда не пересекаются? Нельзя, система уравнений не имеет решения.

Можно также прийти к правильному ответу с помощью метода исключения (также называемого методом сложения или методом линейной комбинации) или методом подстановки.

При использовании метода подстановки мы используем тот факт, что если два выражения y и x имеют одинаковое значение x = y, то x может заменить y или наоборот в другом выражении без изменения значения выражения.


Пример

Решите системы уравнений методом подстановки

$$ \ left \ {\ begin {matrix} y = 2x + 4 \\ y = 3x + 2 \\ \ end {matrix} \ right. $$

Подставляем y в верхнем уравнении выражением для второго уравнения:

$$ \ begin {array} {lcl} 2x + 4 & = & 3x + 2 \\ 4-2 & = & 3x-2x \\ 2 & = & x \\ \ end {array} $$

Чтобы определить значение y , мы можем продолжить, вставив наше значение x в любое из уравнений. Выбираем первое уравнение:

$$ y = 2x + 4 $$

Подключаем x = 2 и получаем

$$ y = 2 \ cdot 2 + 4 = 8 $$

Таким образом, мы пришли к тому же ответу, что и в графическом решении.

Метод исключения требует, чтобы мы добавляли или вычитали уравнения, чтобы исключить x или y , часто нельзя приступить к сложению напрямую, не умножив сначала первое или второе уравнение на некоторое значение.


Пример

$$ 2x-2y = 8 $$

$$ x + y = 1 $$

Теперь мы хотим сложить два уравнения, но это не приведет к исключению x или y .Следовательно, мы должны умножить второе уравнение на 2 с обеих сторон и получить:

$$ 2x-2y = 8 $$

$$ 2x + 2y = 2 $$

Теперь мы пытаемся добавить нашу систему уравнений. Мы начинаем с терминов x слева, а затем с терминов y и, наконец, с цифр справа:

$$ (2x + 2x) + (- 2y + 2y) = 8 + 2 $$

Термины и теперь удалены, и теперь у нас есть уравнение только с одной переменной:

$$ 4x = 10 $$

$$ x = \ frac {10} {4} = 2. 5 $$

После этого, чтобы определить значение y , мы вставляем x = 2,5 в одно из уравнений. Выбираем первое:

$$ \ begin {array} {lcl} 2 \ cdot 2.5-2y & = & 8 \\ 5-8 & = & 2y \\ -3 & = & 2y \\ \ frac {-3} {2} & = & y \\ y & = & -1,5 \\ \ end {array} $$


Видеоурок

Решите систему уравнений:

$$ \ left \ {\ begin {matrix} 2x-4y = 0 \\ -4x + 4y = -4 \ end {matrix} \ right.

$

Линейные системы с двумя переменными

Показать общее уведомление Показать мобильное уведомление Показать все заметки Скрыть все заметки

Это немного заранее, но я хотел сообщить всем, что мои серверы будут проходить техническое обслуживание 17 и 18 мая с 8:00 AM CST до 14:00 PM CST. Будем надеяться, что единственное неудобство будет заключаться в периодическом «потерянном / разорванном» соединении, которое следует исправить, просто перезагрузив страницу. В остальном обслуживание (скрестив пальцы) должно быть «невидимым» для всех.

Пол
6 мая 2021 г.

Похоже, вы используете устройство с «узкой» шириной экрана (, то есть , вероятно, вы используете мобильный телефон). Из-за характера математики на этом сайте лучше всего просматривать в ландшафтном режиме.Если ваше устройство не находится в альбомном режиме, многие уравнения будут отображаться сбоку от вашего устройства (должна быть возможность прокручивать, чтобы увидеть их), а некоторые элементы меню будут обрезаны из-за узкой ширины экрана.

Раздел 7-1: Линейные системы с двумя переменными

Линейная система двух уравнений с двумя переменными — это любая система, которую можно записать в форме.

\ [\ begin {align *} ax + by & = p \\ cx + dy & = q \ end {align *} \]

, где любая из констант может быть равна нулю, за исключением того, что каждое уравнение должно содержать хотя бы одну переменную.

Также система называется линейной, если переменные указаны только в первой степени, присутствуют только в числителе и нет произведений переменных ни в одном из уравнений.

Вот пример системы с числами.

\ [\ begin {align *} 3x — y & = 7 \\ 2x + 3y & = 1 \ end {align *} \]

Прежде чем мы обсудим, как решать системы, мы должны сначала поговорить о том, что такое решение системы уравнений. Решение системы уравнений — это значение \ (x \) и значение \ (y \), которые при подстановке в уравнения удовлетворяют обоим уравнениям одновременно.

В приведенном выше примере \ (x = 2 \) и \ (y = — 1 \) является решением системы. Проверить это достаточно легко.

\ [\ begin {align *} 3 \ left (2 \ right) — \ left ({- 1} \ right) & = 7 \\ 2 \ left (2 \ right) + 3 \ left ({- 1} \ вправо) & = 1 \ end {выровнять *} \]

Итак, конечно, эта пара чисел является решением системы. Не беспокойтесь о том, как мы получили эти ценности. Это будет самая первая система, которую мы решим, когда перейдем к примерам.

Обратите внимание, что важно, чтобы пара чисел удовлетворяла обоим уравнениям. Например, \ (x = 1 \) и \ (y = — 4 \) удовлетворяют первому уравнению, но не второму, и поэтому не являются решением системы. Точно так же \ (x = — 1 \) и \ (y = 1 \) будут удовлетворять второму уравнению, но не первому, и поэтому не могут быть решением системы.

Итак, что же представляет собой решение системы двух уравнений? Хорошо, если вы думаете об этом, оба уравнения в системе являются линиями.Итак, давайте построим их график и посмотрим, что у нас получится.

Как видите, решение системы — это координаты точки пересечения двух линий. Итак, при решении линейных систем с двумя переменными мы действительно спрашиваем, где пересекаются две линии.

В этом разделе мы рассмотрим два метода решения систем.

Первый метод называется методом замены . В этом методе мы решим одно из уравнений для одной из переменных и подставим его в другое уравнение.Это даст одно уравнение с одной переменной, которую мы сможем решить. Как только это решено, мы подставляем это значение обратно в одно из уравнений, чтобы найти значение оставшейся переменной.

На словах этот метод не всегда очень понятен. Давайте рассмотрим пару примеров, чтобы увидеть, как работает этот метод.

Пример 1 Решите каждую из следующих систем.
  1. \ (\ begin {align *} 3x — y & = 7 \\ 2x + 3y & = 1 \ end {align *} \)
  2. \ (\ begin {align *} 5x + 4y & = 1 \\ 3x — 6y & = 2 \ end {align *} \)
Показать все решения Скрыть все решения a \ (\ begin {align *} 3x — y & = 7 \\ 2x + 3y & = 1 \ end {align *} \) Показать решение

Итак, это была первая система, которую мы рассмотрели выше.Мы уже знаем решение, но это даст нам возможность проверить значения, которые мы записали для решения.

Теперь метод говорит, что нам нужно решить одно из уравнений для одной из переменных. Какое уравнение мы выберем и какую переменную выбрать, зависит от вас, но обычно лучше выбрать уравнение и переменную, с которыми будет легко иметь дело. Это означает, что мы должны стараться избегать дробей, если это вообще возможно.

В этом случае, похоже, будет действительно легко решить первое уравнение для \ (y \), так что давайте сделаем это.

\ [3x — 7 = y \]

Теперь подставьте это во второе уравнение.

\ [2x + 3 \ влево ({3x — 7} \ вправо) = 1 \]

Это уравнение в \ (x \), которое мы можем решить, так что давайте сделаем это.

\ [\ begin {align *} 2x + 9x — 21 & = 1 \\ 11x & = 22 \\ x & = 2 \ end {align *} \]

Итак, есть часть решения \ (x \).

Наконец, НЕ забудьте вернуться и найти часть решения \ (y \). Это одна из наиболее распространенных ошибок, которые студенты делают при решении систем. Для этого мы можем либо подставить значение \ (x \) в одно из исходных уравнений и решить для \ (y \), либо просто вставить его в нашу подстановку, которую мы нашли на первом шаге. Так будет проще, так что давайте.

\ [y = 3x — 7 = 3 \ left (2 \ right) — 7 = — 1 \]

Итак, решение — \ (x = 2 \) и \ (y = — 1 \), как мы отметили выше.


b \ (\ begin {align *} 5x + 4y & = 1 \\ 3x — 6y & = 2 \ end {align *} \) Показать решение

С этой системой мы не сможем полностью избежать дробей.Однако похоже, что если мы решим второе уравнение для \ (x \), мы сможем их минимизировать. Вот эта работа.

\ [\ begin {align *} 3x & = 6y + 2 \\ x & = 2y + \ frac {2} {3} \ end {align *} \]

Теперь подставьте это в первое уравнение и решите полученное уравнение относительно \ (y \).

\ [\ begin {align *} 5 \ left ({2y + \ frac {2} {3}} \ right) + 4y & = 1 \\ 10y + \ frac {{10}} {3} + 4y & = 1 \\ 14y & = 1 — \ frac {{10}} {3} = — \ frac {7} {3} \\ y & = — \ left ({\ frac {7} {3}} \ right) \ left ({\ frac {1} {{14}}} \ right) \\ y & = — \ frac {1} {6} \ end {align *} \]

Наконец, подставьте это в исходную замену, чтобы найти \ (x \).

\ [x = 2 \ left ({- \ frac {1} {6}} \ right) + \ frac {2} {3} = — \ frac {1} {3} + \ frac {2} {3} = \ frac {1} {3} \]

Итак, решение этой системы — \ (x = \ frac {1} {3} \) и \ (y = — \ frac {1} {6} \).

Как и в случае с отдельными уравнениями, мы всегда можем вернуться и проверить это решение, подключив его к обоим уравнениям и убедившись, что оно удовлетворяет обоим уравнениям. Также обратите внимание, что нам действительно нужно включить оба уравнения.Вполне возможно, что ошибка может привести к тому, что пара чисел будет удовлетворять одному из уравнений, но не другому.

Теперь перейдем к следующему методу решения систем уравнений. Как мы видели в последней части предыдущего примера, метод подстановки часто заставляет нас иметь дело с дробями, что увеличивает вероятность ошибок. У второго метода этой проблемы не будет. Что ж, это не совсем так. Если будут отображаться дроби, они будут отображаться только на последнем этапе, и они будут отображаться только в том случае, если решение содержит дроби.

Этот второй метод называется методом исключения . В этом методе мы умножаем одно или оба уравнения на соответствующие числа (, т.е. умножаем каждый член в уравнении на число), чтобы одна из переменных имела одинаковый коэффициент с противоположными знаками. Следующим шагом будет сложение двух уравнений. Поскольку одна из переменных имела одинаковый коэффициент с противоположными знаками, она будет удалена, когда мы сложим два уравнения.Результатом будет одно уравнение, которое мы можем решить для одной из переменных. Как только это будет сделано, замените этот ответ на одно из исходных уравнений.

Как и в случае с первым методом, гораздо легче увидеть, что здесь происходит, с помощью пары примеров.

Пример 2 Постановка задачи.
  1. \ (\ begin {align *} 5x + 4y & = 1 \\ 3x — 6y & = 2 \ end {align *} \)
  2. \ (\ begin {align *} 2x + 4y & = — 10 \\ 6x + 3y & = 6 \ end {align *} \)
Показать все решения Скрыть все решения a \ (\ begin {align *} 5x + 4y & = 1 \\ 3x — 6y & = 2 \ end {align *} \) Показать решение

Это система из предыдущего набора примеров, которая заставила нас работать с дробями.Работа с ним здесь покажет различия между двумя методами, а также покажет, что любой метод может использоваться для получения решения для системы.

Итак, нам нужно умножить одно или оба уравнения на константы, чтобы одна из переменных имела одинаковый коэффициент с противоположными знаками. Итак, поскольку члены \ (y \) уже имеют противоположные знаки, давайте работать с этими терминами. Похоже, что если мы умножим первое уравнение на 3, а второе уравнение на 2, члены \ (y \) будут иметь коэффициенты 12 и -12, что нам и нужно для этого метода.

Вот работа для этого шага.

\ [\ begin {align *} 5x + 4y & = 1 & \ underrightarrow {\ times \, \, 3} \ hspace {0.5in} & 15x + 12y = 3 \\ 3x-6y & = 2 & \ underrightarrow {\ times \, \, 2} \ hspace {0,5 дюйма} & \ underline {\, \, 6x-12y = 4} \\ & & & 21x \ hspace {0,5 дюйма} = 7 \\ \ конец {выравнивание *} \]

Итак, как и было обещано в описании метода, у нас есть уравнение, которое можно решить относительно \ (x \).Это дает \ (x = \ frac {1} {3} \), что мы и нашли в предыдущем примере. Однако обратите внимание, что единственная дробь, с которой нам пришлось иметь дело до этого момента, — это сам ответ, который отличается от метода подстановки.

Теперь снова не забудьте найти \ (y \). В этом случае работы будет немного больше, чем метод подстановки. Чтобы найти \ (y \), нам нужно подставить значение \ (x \) в любое из исходных уравнений и решить относительно \ (y \).Поскольку \ (x \) является дробью, заметим, что в этом случае, если мы подставим это значение во второе уравнение, мы потеряем дроби, по крайней мере, временно. Обратите внимание, что часто этого не происходит, и нам придется иметь дело с дробями, хотим мы этого или нет.

\ [\ begin {align *} 3 \ left ({\ frac {1} {3}} \ right) — 6y & = 2 \\ 1 — 6y & = 2 \\ — 6y & = 1 \\ y & = — \ frac {1} {6} \ end {align *} \]

Опять же, это то же значение, которое мы нашли в предыдущем примере.


b \ (\ begin {align *} 2x + 4y & = — 10 \\ 6x + 3y & = 6 \ end {align *} \) Показать решение

В этой части все переменные положительны, поэтому нам придется принудительно установить противоположный знак, умножив где-нибудь на отрицательное число. Также заметим, что в этом случае, если мы просто умножим первое уравнение на -3, то коэффициенты при \ (x \) будут -6 и 6.

Иногда нам нужно только умножить одно из уравнений, а другое можно оставить в покое.Вот эта работа по этой части.

\ [\ begin {align *} 2x + 4y & = -10 & \ underrightarrow {\ times \, \, — 3} \ hspace {0,5 дюйма} & -6x-12y = 30 \\ 6x + 3y & = 6 & \ underrightarrow {\ text {same}} \ hspace {0,5 дюйма} & \ underline {\ hspace {0,35 дюйма} 6x + 3y = 6} \\ & & & \ hspace {0,5 дюйма} -9y = 36 \\ & & & \ hspace {0,85 дюйма} y = -4 \\ \ конец {выравнивание *} \]

Наконец, подставьте это в любое из уравнений и решите относительно \ (x \).На этот раз мы воспользуемся первым уравнением.

\ [\ begin {align *} 2x + 4 \ left ({- 4} \ right) & = — 10 \\ 2x — 16 & = — 10 \\ 2x & = 6 \\ x & = 3 \ end {align *} \]

Итак, решение этой системы — \ (x = 3 \) и \ (y = — 4 \).

Существует третий метод, который мы рассмотрим для решения систем из двух уравнений, но он немного сложнее и, вероятно, более полезен для систем, по крайней мере, с тремя уравнениями, поэтому мы рассмотрим его в следующем разделе.

Перед тем, как покинуть этот раздел, мы должны рассмотреть несколько частных случаев решения систем.

Пример 3 Решите следующие системы уравнений. \ [\ begin {align *} x — y & = 6 \\ — 2x + 2y & = 1 \ end {align *} \] Показать решение

Здесь мы можем использовать любой метод, но похоже, что замена будет немного проще. Мы решим первое уравнение относительно \ (x \) и подставим его во второе уравнение.

\ [\ begin {align *} x & = 6 + y \\ & \\ — 2 \ left ({6 + y} \ right) + 2y & = 1 \\ — 12 — 2y + 2y & = 1 \\ — 12 & = 1 \, \, \, ?? \ end {align *} \]

Итак, это явно неправда, и, похоже, нигде в нашей работе нет ошибки. Так в чем проблема? Чтобы увидеть, давайте изобразим эти две линии и посмотрим, что мы получим.

Похоже, что эти две прямые параллельны (можете ли вы проверить это с помощью наклона?), И мы знаем, что две параллельные линии с разными пересечениями \ (y \) (что важно) никогда не пересекутся.

Как мы видели в начале обсуждения этого раздела, решения представляют собой точку пересечения двух линий. Если две линии не пересекаются, у нас не будет решения.

Итак, когда мы получаем такой бессмысленный ответ в результате нашей работы, у нас есть две параллельные линии и нет решения этой системы уравнений.

Система в предыдущем примере называется несовместимая .Также обратите внимание, что если бы мы использовали исключение в этой системе, мы бы получили аналогичный бессмысленный ответ.

Пример 4 Решите следующую систему уравнений. \ [\ begin {align *} 2x + 5y & = — 1 \\ — 10x — 25y & = 5 \ end {align *} \] Показать решение

В этом примере кажется, что устранение будет самым простым методом.

\ [\ begin {align *} 2x + 5y & = -1 & \ underrightarrow {\ times \, \, 5} \ hspace {0.5in} & \, \, \, \, 10x + 25y = -5 \\ -10x-25y & = 5 & \ underrightarrow {\ text {same}} \ hspace {0,5 дюйма} & \ underline {-10x-25y = 5} \\ & & & \ hspace {0.9in} 0 = 0 \\ \ конец {выравнивание *} \]

На первый взгляд может показаться, что это та же проблема, что и в предыдущем примере. Однако в этом случае мы пришли к равенству, которое просто не соответствовало действительности. В этом случае мы имеем 0 = 0, и это истинное равенство, и в этом смысле в этом нет ничего плохого.

Однако это явно не тот ответ, который мы ожидали здесь, и поэтому нам нужно определить, что именно происходит.

Мы предоставим вам возможность проверить это, но если вы найдете наклон и \ (y \) — точки пересечения для этих двух линий, вы обнаружите, что обе линии имеют точно такой же наклон, и обе линии имеют точно такой же \ ( y \) — перехват. Итак, что это значит для нас? Хорошо, если две линии имеют одинаковый наклон и одинаковые \ (y \) — точки пересечения, тогда графики этих двух линий являются одним и тем же графиком.Другими словами, графики этих двух линий — это один и тот же график. В этих случаях любой набор точек, удовлетворяющий одному из уравнений, также будет удовлетворять другому уравнению.

Также напомним, что график уравнения — это не что иное, как набор всех точек, удовлетворяющих уравнению. Другими словами, существует бесконечный набор точек, которые удовлетворяют этой системе уравнений.

В этих случаях мы действительно хотим записать что-нибудь для решения.Итак, что мы сделаем, так это решим одно из уравнений для одной из переменных (неважно, что вы выберете). Решим первую относительно \ (y \).

\ [\ begin {align *} 2x + 5y & = — 1 \\ 5y & = — 2x — 1 \\ y & = — \ frac {2} {5} x — \ frac {1} {5} \ end {выровнять*}\]

Затем для любого \ (x \) мы можем найти \ (y \), и эти два числа образуют решение системы уравнений. Обычно мы обозначаем это, записывая решение следующим образом:

\ [\ begin {array} {* {20} {c}} \ begin {align} x & = t \\ y & = — \ frac {2} {5} t — \ frac {1} {5} \ конец {выровнен} & {\ hspace {0.25in} {\ mbox {где}} \, t {\ mbox {- любое действительное число}}} \ end {array} \]

Чтобы показать, что они дают решения, давайте рассмотрим пару значений \ (t \).

\ (t = 0 \)

\ [x = 0 \ hspace {0,25 дюйма} y = — \ frac {1} {5} \]

Чтобы показать, что это решение, нам нужно вставить его в оба уравнения системы.

\ [\ begin {align *} 2 \ left (0 \ right) + 5 \ left ({- \ frac {1} {5}} \ right) & \ mathop = \ limits ^? — 1 & \ hspace {0. ? 5 \\ — 1 & = — 1 & \ hspace {0,25 дюйма} 5 & = 5 \ end {align *} \]

Итак, \ (x = 0 \) и \ (y = — \ frac {1} {5} \) является решением системы. Давай быстро сделаем еще один.

\ (t = — 3 \)

\ [x = — 3 \ hspace {0,25 дюйма} y = — \ frac {2} {5} \ left ({- 3} \ right) — \ frac {1} {5} = \ frac {6} {5 } — \ frac {1} {5} = 1 \]

И снова нам нужно вставить его в оба уравнения системы, чтобы показать, что это решение.? 5 \\ — 1 & = — 1 & \ hspace {0,25 дюйма} 5 & = 5 \ end {align *} \]

Конечно, \ (x = — 3 \) и \ (y = 1 \) — это решение.

Итак, поскольку существует бесконечное количество возможных \ (t \) ‘, должно быть бесконечное количество решений для этой системы, и они даются как,

\ [\ begin {array} {* {20} {c}} \ begin {align} x & = t \\ y & = — \ frac {2} {5} t — \ frac {1} {5} \ конец {выровнен} & {\ hspace {0. 25in} {\ mbox {где}} \, t {\ mbox {- любое действительное число}}} \ end {array} \]

Системы, подобные тем, что в предыдущих примерах, называются зависимыми .

Теперь мы увидели все три возможности решения системы уравнений. Система уравнений не будет иметь решения, ровно одно решение или бесконечно много решений.

4.1: Решение систем линейных уравнений с двумя переменными

Определить, является ли упорядоченная пара решением системы уравнений

В разделе Решение линейных уравнений мы узнали, как решать линейные уравнения с одной переменной.Теперь мы будем работать с двумя или более линейными уравнениями, сгруппированными вместе, что известно как система линейных уравнений .

СИСТЕМА ЛИНЕЙНЫХ УРАВНЕНИЙ

Когда два или более линейных уравнения сгруппированы вместе, они образуют систему линейных уравнений .

В этом разделе мы сосредоточим нашу работу на системах двух линейных уравнений с двумя неизвестными. Позже в этой главе мы решим более крупные системы уравнений.

Пример системы двух линейных уравнений показан ниже.Мы используем скобку, чтобы показать, что два уравнения сгруппированы вместе и образуют систему уравнений.

\ [\ left \ {\ begin {выровнено} 2x + y & = 7 \\ x − 2y & = 6 \ end {выровнено} \ right. \ nonumber \]

Линейное уравнение с двумя переменными, например \ (2x + y = 7 \), имеет бесконечное число решений. Его график представляет собой линию. Помните, что каждая точка на линии — это решение уравнения, а каждое решение уравнения — это точка на линии.

Чтобы решить систему двух линейных уравнений, мы хотим найти значения переменных, которые являются решениями обоих уравнений.Другими словами, мы ищем упорядоченные пары \ ((x, y) \), которые делают оба уравнения истинными. Они называются решениями системы уравнений .

РЕШЕНИЯ СИСТЕМЫ УРАВНЕНИЙ

решений системы уравнений — это значения переменных, которые делают все уравнениями истинными. Решение системы двух линейных уравнений представляется упорядоченной парой \ ((x, y) \).

Чтобы определить, является ли упорядоченная пара решением системы двух уравнений, мы подставляем значения переменных в каждое уравнение.Если упорядоченная пара делает оба уравнения истинными, это решение системы.

Пример \ (\ PageIndex {1} \)

Определите, является ли упорядоченная пара решением системы \ (\ left \ {\ begin {array} {l} x − y = −1 \\ 2x − y = −5 \ end {array} \ right. \) .

ⓐ \ ((- 2, −1) \) ⓑ \ ((- 4, −3) \)

Ответ

Пример \ (\ PageIndex {2} \)

Определите, является ли упорядоченная пара решением системы \ (\ left \ {\ begin {array} 3x + y = 0 \\ x + 2y = −5 \ end {array} \ right.\).

ⓐ \ ((1, −3) \) ⓑ \ ((0,0) \)

Ответ

ⓐ да ⓑ нет

Пример \ (\ PageIndex {3} \)

Определите, является ли упорядоченная пара решением системы \ (\ left \ {\ begin {array} x − 3y = −8 \\ −3x − y = 4 \ end {array} \ right. \).

ⓐ \ ((2, −2) \) ⓑ \ ((- 2,2) \)

Ответ

ⓐ нет ⓑ да

Решите систему линейных уравнений с помощью построения графиков

В этом разделе мы будем использовать три метода для решения системы линейных уравнений.Первый метод, который мы будем использовать, — это построение графиков.

График линейного уравнения представляет собой линию. Каждая точка на линии — это решение уравнения. Для системы двух уравнений мы построим график двумя линиями. Затем мы можем увидеть все точки, которые являются решениями каждого уравнения. И, обнаружив, что общего у линий, мы найдем решение системы.

Большинство линейных уравнений с одной переменной имеют одно решение, но мы видели, что некоторые уравнения, называемые противоречиями, не имеют решений, а для других уравнений, называемых тождествами, все числа являются решениями.

Точно так же, когда мы решаем систему двух линейных уравнений, представленную графиком из двух линий в одной плоскости, есть три возможных случая, как показано.

Рисунок \ (\ PageIndex {1} \)

Каждый раз, когда мы демонстрируем новый метод, мы будем использовать его в той же системе линейных уравнений. В конце раздела вы решите, какой метод был наиболее удобным для решения этой системы.

Пример \ (\ PageIndex {4} \): как решить систему уравнений с помощью построения графиков

Решите систему, построив график \ (\ left \ {\ begin {array} {l} 2x + y = 7 \\ x − 2y = 6 \ end {array} \ right.\).

Ответ

Пример \ (\ PageIndex {5} \)

Решите систему, построив график: \ (\ left \ {\ begin {array} {l} x − 3y = −3 \\ x + y = 5 \ end {array} \ right. \).

Ответ

\ ((3,2) \)

Пример \ (\ PageIndex {6} \)

Решите систему, построив график: \ (\ left \ {\ begin {array} {l} −x + y = 1 \\ 3x + 2y = 12 \ end {array} \ right.\)

Ответ

\ ((2,3) \)

Здесь показаны шаги, которые необходимо использовать для решения системы линейных уравнений с помощью построения графиков.

РЕШИТЕ ​​СИСТЕМУ ЛИНЕЙНЫХ УРАВНЕНИЙ С ПОМОЩЬЮ ГРАФИКОВ.

  1. Изобразите первое уравнение.
  2. Постройте второе уравнение в той же прямоугольной системе координат.
  3. Определите, пересекаются ли линии, параллельны или совпадают.
  4. Определите решение системы.
    • Если линии пересекаются, укажите точку пересечения. Это решение системы.
    • Если линии параллельны, у системы нет решения.
    • Если линии совпадают, система имеет бесконечное количество решений.
  5. Проверьте решение в обоих уравнениях.

В следующем примере мы сначала перепишем уравнения в форме углового пересечения, так как это упростит нам быстрое построение графика линий.

Пример \ (\ PageIndex {7} \)

Решите систему, построив график: \ (\ left \ {\ begin {array} {l} 3x + y = −1 \\ 2x + y = 0 \ end {array} \ right. \)

Ответ

Мы решим оба этих уравнения относительно \ (y \), чтобы мы могли легко построить их график, используя их наклоны и \ (y \) — точки пересечения.

Пример \ (\ PageIndex {8} \)

Решите систему, построив график: \ (\ left \ {\ begin {array} {l} −x + y = 1 \\ 2x + y = 10 \ end {array} \ right.\).

Ответ

\ ((3,4) \)

Пример \ (\ PageIndex {9} \)

Решите систему, построив график: \ (\ left \ {\ begin {array} {l} 2x + y = 6 \\ x + y = 1 \ end {array} \ right. \).

Ответ

\ ((5, −4) \)

До сих пор во всех системах линейных уравнений линии пересекались и решение было одной точкой. В следующих двух примерах мы рассмотрим систему уравнений, не имеющую решения, и систему уравнений, которая имеет бесконечное число решений.

Пример \ (\ PageIndex {10} \)

Решите систему, построив график: \ (\ left \ {\ begin {array} {l} y = \ tfrac {1} {2} x-3 \\ x-2y = 4 \ end {array} \ right. \ ).

Ответ

Пример \ (\ PageIndex {11} \)

Решите систему, построив график: \ (\ left \ {\ begin {array} {l} y = — \ tfrac {1} {4} x + 2 \\ x + 4y = 4 \ end {array} \ right. \).

Ответ

нет решения

Пример \ (\ PageIndex {12} \)

Решите систему, построив график: \ (\ left \ {\ begin {array} {l} y = 3x-1 \\ 6x-2y = 6 \ end {array} \ right.\).

Ответ

нет решения

Иногда уравнения в системе представляют собой одну и ту же линию. Поскольку каждая точка на прямой делает оба уравнения истинными, существует бесконечно много упорядоченных пар, которые делают оба уравнения истинными. У системы бесконечно много решений.

Пример \ (\ PageIndex {13} \)

Решите систему, построив график: \ (\ left \ {\ begin {array} {l} y = 2x-3 \\ -6x + 3y = 9 \ end {array} \ right.\).

Ответ

Если вы напишете второе уравнение в форме пересечения наклона, вы можете заметить, что уравнения имеют одинаковый наклон и одинаковое пересечение y .

Пример \ (\ PageIndex {14} \)

Решите систему, построив график: \ (\ left \ {\ begin {array} {l} y = -3x-6 \\ 6x + 2y = -12 \ end {array} \ right. \).

Ответ

бесконечно много решений

Пример \ (\ PageIndex {15} \)

Решите систему, построив график: \ (\ left \ {\ begin {array} {l} y = \ tfrac {1} {2} x-4 \\ 2x-4y = 16 \ end {array} \ right.\).

Ответ

бесконечно много решений

Когда мы нарисовали вторую линию в последнем примере, мы нарисовали ее прямо над первой линией. Мы говорим, что две строки совпадают с . Совпадающие линии имеют одинаковый наклон и точку пересечения y-.

СОВПАДАЮЩИЕ ЛИНИИ

Совпадающие линии имеют одинаковый наклон и одинаковую точку пересечения y- .

Каждая из систем уравнений в примере и примере имела две пересекающиеся линии.У каждой системы было одно решение.

В примере уравнения давали совпадающие линии, поэтому система имела бесконечно много решений.

Системы в этих трех примерах имели по крайней мере одно решение. Система уравнений, имеющая по крайней мере одно решение, называется согласованной системой .

Система с параллельными линиями, такая как Пример , не имеет решения. Мы называем такую ​​систему уравнений несогласованной. Нет решения.

СОГЛАСОВАННЫЕ И НЕПОСРЕДСТВЕННЫЕ СИСТЕМЫ

Согласованная система уравнений — это система уравнений, имеющая по крайней мере одно решение.

Непоследовательная система уравнений — это система уравнений, не имеющая решения.

Мы также классифицируем уравнения в системе уравнений, называя уравнения независимыми или зависимыми . Если два уравнения независимы, каждое из них имеет собственный набор решений.Пересекающиеся линии и параллельные линии независимы.

Если два уравнения являются зависимыми, все решения одного уравнения также являются решениями другого уравнения. Когда мы строим график двух зависимых уравнений, мы получаем совпадающие линии.

Подведем итог, посмотрев на графики трех типов систем. См. Ниже и таблицу .

Строки Пересечение Параллельный Совпадение
Количество решений 1 балл Нет решения Бесконечно много
Согласованный / непоследовательный Согласованный Несоответствие Согласованный
Зависимые / независимые Независимая Независимая Зависимые

Пример \ (\ PageIndex {16} \)

Без построения графиков определите количество решений, а затем классифицируйте систему уравнений.

ⓐ \ (\ left \ {\ begin {array} {l} y = 3x − 1 \\ 6x − 2y = 12 \ end {array} \ right. \) Ⓑ \ (\ left \ {\ begin {array} {l} 2x + y = −3 \\ x − 5y = 5 \ end {array} \ right. \)

Ответ

ⓐ Сравним наклоны и пересечения двух линий.

\ (\ begin {array} {lll} {} & {} & {\ left \ {\ begin {array} {l} {y = 3x-1} \\ {6x − 2y = 12} \ end {массив } \ right.} \\ {} & {} & {y = 3x-1} \\ {\ text {Первое уравнение уже находится в форме пересечения наклона.}} & {} & {} \\ {\ text {Запишите второе уравнение в форме пересечения наклона.}} & {} & {} \\ {} & {} & {} \\ {} & {} & {} \\ {} & {} & {} \\ {} & {} & {6x-2y = 12} \\ {} & {} & {- 2y = -6x + 12} \\ {} & { } & {\ frac {-2y} {- 2} = \ frac {-6x + 12} {- 2}} \\ {} & {} & {y = 3x-6} \\ {} & {y = 3x-1} & {y = 3x-6} \\ {} & {m = 3} & {m = 3} \\ {} & {b = -1} & {b = -6} \\ {\ text {Найдите наклон и точку пересечения каждой линии.}} & {} & {} \\ {} & {} & {} \\ {} & {} & {} \\ {} & {} & {} \ \ {} & {} & {} \\ {} & {\ text {Поскольку наклоны одинаковые, а точки пересечения y}} & {} \\ {} & {\ text {разные, линии параллельны.}} & {} \\ \ end {array} \)

ⓑ Мы сравним наклон и пересечения двух линий.

\ (\ begin {array} {lll} {} & {} & {} \\ {} & {\ left \ {\ begin {array} {l} 2x + y = -3 \\ x-5y = 5 \\ \ end {array} \ right.} & {} \\ {\ text {Запишите оба уравнения в форме углового пересечения.}} & {} & {} \\ {} & {} & {} \\ { } & {} & {} \\ {} & {} & {} \\ {} & {2x + y = -3} & {x-5y = 5} \\ {} & {y = -2x-3 } & {- 5y = -x + 5} \\ {} & {} & {\ frac {-5y} {- 5} = \ frac {-x + 5} {- 5}} \\ {} & { } & {y = \ frac {1} {5} -1} \\ {} & {} & {} \\ {} & {} & {} \\ {} & {} & {} \\ {} & {} & {} \\ {\ text {Найдите наклон и точку пересечения каждой линии.}} & {} & {} \\ {} & {} & {} \\ {} & {y = -2x-3} & {y = \ frac {1} {5} -1} \\ {} & {m = -2} & {m = \ frac {1} {5}} \\ {} & {b = -3} & {b = -1} \\ {} & {} & {} \\ {} & {\ text {Поскольку уклоны разные, линии пересекаются.}} & {} \\ \ end {array} \)

Система уравнений, графики которой пересекаются, имеет одно решение, непротиворечива и независима.

Пример \ (\ PageIndex {17} \)

Без построения графиков определите количество решений, а затем классифицируйте систему уравнений.

ⓐ \ (\ left \ {\ begin {array} {l} y = −2x − 4 \\ 4x + 2y = 9 \ end {array} \ right. \) Ⓑ \ (\ left \ {\ begin {array } {l} 3x + 2y = 2 \\ 2x + y = 1 \ end {array} \ right. \)

Ответ

ⓐ нет решения, непоследовательное, независимое ⓑ одно решение, последовательное, независимое

Пример \ (\ PageIndex {18} \)

Без построения графиков определите количество решений, а затем классифицируйте систему уравнений.

ⓐ \ (\ left \ {\ begin {array} {l} y = \ frac {1} {3} x − 5 \\ x − 3y = 6 \ end {array} \ right.\) Ⓑ \ (\ left \ {\ begin {array} {l} x + 4y = 12 \\ −x + y = 3 \ end {array} \ right. \)

Ответ

ⓐ нет решения, непоследовательное, независимое ⓑ одно решение, последовательное, независимое

Решение систем линейных уравнений с помощью графиков — хороший способ визуализировать типы решений, которые могут возникнуть. Однако во многих случаях решение системы с помощью построения графиков неудобно или неточно. Если графики выходят за пределы небольшой сетки с x и y как между \ (- 10 \), так и 10, построение линий может быть громоздким.И если решения системы не являются целыми числами, может быть трудно точно прочитать их значения с графика.

Решите систему уравнений подстановкой

Теперь решим системы линейных уравнений методом подстановки.

Мы будем использовать ту же систему, которую мы использовали вначале для построения графиков.

\ [\ left \ {\ begin {array} {l} 2x + y = 7 \\ x − 2y = 6 \ end {array} \ right. \ nonumber \]

Сначала мы решим одно из уравнений относительно x или y .Мы можем выбрать любое уравнение и решить любую переменную, но мы постараемся сделать выбор, который упростит работу.

Затем мы подставляем это выражение в другое уравнение. В результате получается уравнение с одной переменной — и мы знаем, как его решить!

После того, как мы найдем значение одной переменной, мы подставим это значение в одно из исходных уравнений и решим для другой переменной. Наконец, мы проверяем наше решение и убеждаемся, что оно выполняет оба уравнения.

Пример \ (\ PageIndex {20} \)

Решите систему заменой: \ (\ left \ {\ begin {array} {l} −2x + y = −11 \\ x + 3y = 9 \ end {array} \ right. \)

Ответ

\ ((6,1) \)

Пример \ (\ PageIndex {21} \)

Решите систему заменой: \ (\ left \ {\ begin {array} {l} 2x + y = −1 \\ 4x + 3y = 3 \ end {array} \ right. \)

Ответ

\ ((- 3,5) \)

РЕШИТЬ СИСТЕМУ УРАВНЕНИЙ ПОДСТАВКОЙ.

  1. Решите одно из уравнений для любой переменной.
  2. Подставьте выражение из шага 1 в другое уравнение.
  3. Решите полученное уравнение.
  4. Подставьте решение шага 3 в любое из исходных уравнений, чтобы найти другую переменную.
  5. Запишите решение в виде упорядоченной пары.
  6. Убедитесь, что упорядоченная пара является решением обоих исходных уравнений.

Будьте очень осторожны со знаками в следующем примере.

Пример \ (\ PageIndex {22} \)

Решите систему заменой: \ (\ left \ {\ begin {array} {l} 4x + 2y = 4 \\ 6x − y = 8 \ end {array} \ right. \)

Ответ

Нам нужно решить одно уравнение для одной переменной. Решим первое уравнение относительно y .

Пример \ (\ PageIndex {23} \)

Решите систему заменой: \ (\ left \ {\ begin {array} {l} x − 4y = −4 \\ −3x + 4y = 0 \ end {array} \ right.\)

Ответ

\ ((2,32) \)

Пример \ (\ PageIndex {24} \)

Решите систему заменой: \ (\ left \ {\ begin {array} {l} 4x − y = 0 \\ 2x − 3y = 5 \ end {array} \ right. \)

Ответ

\ ((- 12, −2) \)

Решите систему уравнений методом исключения

Мы решили системы линейных уравнений с помощью построения графиков и подстановки.Построение графиков хорошо работает, когда переменные коэффициенты малы, а решение имеет целочисленные значения. Подстановка работает хорошо, когда мы можем легко решить одно уравнение для одной из переменных и не иметь слишком много дробей в результирующем выражении.

Третий метод решения систем линейных уравнений называется методом исключения. Когда мы решали систему с помощью подстановки, мы начинали с двух уравнений и двух переменных и сводили ее к одному уравнению с одной переменной. То же самое мы сделаем и с методом исключения, но у нас будет другой способ добиться этого.

Метод исключения основан на добавочном свойстве равенства. Свойство сложения равенства говорит, что когда вы добавляете одинаковую величину к обеим сторонам уравнения, вы все равно получаете равенство. Мы расширим свойство сложения равенства, чтобы сказать, что когда вы добавляете равные количества к обеим сторонам уравнения, результаты равны.

Для любых выражений a, b, c, и d .

\ [\ begin {array} {ll} {\ text {if}} & {a = b} \\ {\ text {and}} & {c = d} \\ {\ text {then}} & { а + с = б + г.} \\ \ nonumber \ end {array} \]

Чтобы решить систему уравнений методом исключения, мы начнем с обоих уравнений в стандартной форме. Затем мы решаем, какую переменную будет легче всего устранить. Как мы решаем? Мы хотим, чтобы коэффициенты одной переменной были противоположными, чтобы мы могли сложить уравнения и исключить эту переменную.

Обратите внимание, как это работает, когда мы складываем эти два уравнения вместе:

\ [\ left \ {\ begin {array} {l} 3x + y = 5 \\ \ underline {2x − y = 0} \ end {array} \ right.\ nonumber \]

\ [5x = 5 \ nonumber \]

и прибавляют к нулю, и мы получаем одно уравнение с одной переменной.

Давайте попробуем еще один:

\ [\ left \ {\ begin {array} x + 4y = 2 \\ 2x + 5y = −2 \ end {array} \ right. \ nonumber \]

На этот раз мы не видим переменную, которая может быть немедленно удалена, если мы добавим уравнения.

Но если мы умножим первое уравнение на \ (- 2 \), мы сделаем коэффициенты x противоположными.Мы должны умножить каждый член в обеих частях уравнения на \ (- 2 \).

Затем перепишите систему уравнений.

Теперь мы видим, что коэффициенты членов x противоположны, поэтому x будет исключено, когда мы сложим эти два уравнения.

Как только мы получаем уравнение с одной переменной, мы его решаем. Затем мы подставляем это значение в одно из исходных уравнений, чтобы найти оставшуюся переменную.И, как всегда, мы проверяем наш ответ, чтобы убедиться, что он является решением обоих исходных уравнений.

Теперь мы увидим, как использовать исключение для решения той же системы уравнений, которую мы решили с помощью построения графиков и подстановки.

Упражнение \ (\ PageIndex {26} \)

Решите систему методом исключения: \ (\ left \ {\ begin {array} {l} 3x + y = 5 \\ 2x − 3y = 7 \ end {array} \ right. \)

Ответ

\ ((2, −1) \)

Упражнение \ (\ PageIndex {27} \)

Решите систему методом исключения: \ (\ left \ {\ begin {array} {l} 4x + y = −5 \\ ​​−2x − 2y = −2 \ end {array} \ right.\)

Ответ

\ ((- 2,3) \)

Шаги перечислены здесь для удобства.

РЕШИТЬ СИСТЕМУ УРАВНЕНИЙ ПУТЕМ ИСКЛЮЧЕНИЯ.

  1. Запишите оба уравнения в стандартной форме. Если какие-либо коэффициенты являются дробными, очистите их.
  2. Сделайте коэффициенты одной переменной противоположными.
    • Решите, какую переменную исключить.
    • Умножьте одно или оба уравнения так, чтобы коэффициенты этой переменной были противоположными.
  3. Добавьте уравнения, полученные на шаге 2, чтобы исключить одну переменную.
  4. Найдите оставшуюся переменную.
  5. Подставьте решение из шага 4 в одно из исходных уравнений. Затем найдите другую переменную.
  6. Запишите решение в виде упорядоченной пары.
  7. Убедитесь, что упорядоченная пара является решением обоих исходных уравнений.

Теперь мы рассмотрим пример, в котором нам нужно умножить оба уравнения на константы, чтобы сделать коэффициенты одной переменной противоположными.

Упражнение \ (\ PageIndex {28} \)

Решите систему методом исключения: \ (\ left \ {\ begin {array} {l} 4x − 3y = 9 \\ 7x + 2y = −6 \ end {array} \ right. \)

Ответ

В этом примере мы не можем умножить одно уравнение на любую константу, чтобы получить противоположные коэффициенты. Поэтому мы стратегически умножим оба уравнения на разные константы, чтобы получить противоположности.

Упражнение \ (\ PageIndex {29} \)

Решите систему методом исключения: \ (\ left \ {\ begin {array} {l} 3x − 4y = −9 \\ 5x + 3y = 14 \ end {array} \ right.\)

Ответ

\ ((1,3) \)

Упражнение \ (\ PageIndex {30} \)

Решите каждую систему методом исключения: \ (\ left \ {\ begin {array} {l} 7x + 8y = 4 \\ 3x − 5y = 27 \ end {array} \ right. \)

Ответ

\ ((4, −3) \)

Когда система уравнений содержит дроби, мы сначала очистим дроби, умножив каждое уравнение на ЖК-дисплей всех дробей в уравнении.

Упражнение \ (\ PageIndex {31} \)

Решите систему методом исключения: \ (\ left \ {\ begin {array} {l} x + \ tfrac {1} {2} y = 6 \\ \ tfrac {3} {2} x + \ tfrac {2} { 3} y = \ tfrac {17} {2} \ end {array} \ right. \)

Ответ

В этом примере в обоих уравнениях есть дроби. Нашим первым шагом будет умножение каждого уравнения на ЖК-дисплей всех дробей в уравнении, чтобы очистить дроби.

Упражнение \ (\ PageIndex {32} \)

Решите каждую систему методом исключения: \ (\ left \ {\ begin {array} {l} \ tfrac {1} {3} x− \ tfrac {1} {2} y = 1 \\ \ tfrac {3} { 4} x − y = \ tfrac {5} {2} \ end {array} \ right.\)

Ответ

\ ((6,2) \)

Упражнение \ (\ PageIndex {33} \)

Решите каждую систему методом исключения: \ (\ left \ {\ begin {array} {l} x + \ tfrac {3} {5} y = — \ tfrac {1} {5} \\ — \ tfrac {1} { 2} x− \ tfrac {2} {3} y = \ tfrac {5} {6} \ end {array} \ right. \)

Ответ

\ ((1, −2) \)

Когда мы решили систему с помощью построения графиков, мы увидели, что не все системы линейных уравнений имеют единственную упорядоченную пару в качестве решения.Когда два уравнения действительно представляли собой одну и ту же линию, решений было бесконечно много. Мы назвали это последовательной системой. Когда два уравнения описывали параллельные линии, решения не было. Мы назвали это несовместимой системой.

То же самое и с заменой или исключением. Если уравнение в конце замены или исключения является истинным утверждением, у нас есть непротиворечивая, но зависимая система, а система уравнений имеет бесконечно много решений. Если уравнение в конце замены или исключения является ложным утверждением, мы имеем несовместимую систему и система уравнений не имеет решения.

Упражнение \ (\ PageIndex {34} \)

Решите систему методом исключения: \ (\ left \ {\ begin {array} {l} 3x + 4y = 12 \\ y = 3− \ tfrac {3} {4} x \ end {array} \ right. \ )

Ответ

\ (\ begin {array} {ll} {} & {\ left \ {\ begin {array} {l} 3x + 4y = 12 \\ y = 3− \ frac {3} {4} x \ end { array} \ right.} \\ {} & {} \\ {\ text {Запишите второе уравнение в стандартной форме.}} & {\ left \ {\ begin {array} {l} 3x + 4y = 12 \\ \ frac {3} {4} x + y = 3 \ end {array} \ right.} \\ {} & {} \\ {\ text {Очистите дроби, умножив} \\ \ text {второе уравнение на 4.}} & {\ left \ {\ begin {array} {l} 3x + 4y = 12 \\ 4 (\ frac {3} {4} x + y) = 4 (3) \ end {array} \ right. } \\ {} & {} \\ {\ text {Упростить.}} & {\ left \ {\ begin {array} {l} 3x + 4y = 12 \\ 3x + 4y = 12 \ end {array} \ верно. } \\ {} & {} \\ {\ text {Чтобы исключить переменную, мы умножаем второе уравнение} \\ \ text {на -1. Упростите и добавьте.}} & {\ Begin {array} {l} {\ left \ {\ begin {array} {l} 3x + 4y = 12 \\ \ underline {-3x-4y = -12} \ end { массив} \ право.} \\ {\ hspace {16mm} 0 = 0} \ end {array}} \\ \ end {array} \)

Это верное заявление. Уравнения непротиворечивы, но зависимы. Их графики будут одной линией. У системы бесконечно много решений.

После того, как мы очистили дроби во втором уравнении, заметили ли вы, что эти два уравнения совпадают? Это означает, что у нас есть совпадающие линии.

Упражнение \ (\ PageIndex {35} \)

Решите систему методом исключения: \ (\ left \ {\ begin {array} {l} 5x − 3y = 15 \\ 5y = −5 + \ tfrac {5} {3} x \ end {array} \ right.\)

Ответ

бесконечно много решений

Упражнение \ (\ PageIndex {36} \)

Решите систему путем исключения: \ (\ left \ {\ begin {array} {l} x + 2y = 6 \\ y = — \ tfrac {1} {2} x + 3 \ end {array} \ right. \)

Ответ

бесконечно много решений

Выберите наиболее удобный метод решения системы линейных уравнений

Когда вы решаете систему линейных уравнений в приложении, вам не сообщат, какой метод использовать.Вам нужно будет принять это решение самостоятельно. Так что вы захотите выбрать самый простой метод, который сводит к минимуму ваши шансы на ошибку.

\ [\ textbf {Выберите наиболее удобный метод для решения системы линейных уравнений} \\ \ begin {array} {lll} {\ underline {\ textbf {Graphing}}} & {\ underline {\ textbf {Substitution} }} & {\ underline {\ textbf {Исключение}}} \\ {\ text {Используется, когда вам нужно}} & {\ text {Используется, когда одно уравнение равно}} & {\ text {Используется, когда уравнения a} } \\ {\ text {картина ситуации.}} & {\ text {уже решено или может быть}} & {\ text {переустановить стандартную форму.}} \\ {\ text {}} & {\ text {легко решено для одного}} & {\ text {} } \\ {\ text {}} & {\ text {переменная.}} & {\ text {}} \\ \ end {array} \ nonumber \]

Пример \ (\ PageIndex {37} \)

Решите для каждой системы линейных уравнений, что удобнее решить: заменой или исключением. Поясните свой ответ.

ⓐ \ (\ left \ {\ begin {array} {l} 3x + 8y = 40 \\ 7x − 4y = −32 \ end {array} \ right.\) Ⓑ \ (\ left \ {\ begin {array} {l} 5x + 6y = 12 \\ y = \ tfrac {2} {3} x − 1 \ end {array} \ right. \)

Ответ

\ [\ left \ {\ begin {array} {l} 3x + 8y = 40 \\ 7x − 4y = −32 \ end {array} \ right. \ Nonumber \]

Поскольку оба уравнения имеют стандартную форму, использование исключения будет наиболее удобным.

\ [\ left \ {\ begin {array} {l} 5x + 6y = 12 \\ y = \ tfrac {2} {3} x − 1 \ end {array} \ right. \ Nonumber \]

Поскольку одно уравнение уже решено относительно y , использование подстановки будет наиболее удобным.

Пример \ (\ PageIndex {38} \)

Для каждой системы линейных уравнений решите, что удобнее будет решить заменой или исключением. Поясните свой ответ.

ⓐ \ (\ left \ {\ begin {array} {l} 4x − 5y = −32 \\ 3x + 2y = −1 \ end {array} \ right. \) Ⓑ \ (\ left \ {\ begin { массив} {l} x = 2y − 1 \\ 3x − 5y = −7 \ end {array} \ right. \)

Ответ

ⓐ Поскольку оба уравнения имеют стандартную форму, использование исключения будет наиболее удобным.Ⓑ Поскольку одно уравнение уже решено для x , использование подстановки будет наиболее удобным.

Пример \ (\ PageIndex {39} \)

Для каждой системы линейных уравнений решите, что удобнее будет решить заменой или исключением. Поясните свой ответ.

ⓐ \ (\ left \ {\ begin {array} {l} y = 2x − 1 \\ 3x − 4y = −6 \ end {array} \ right. \) Ⓑ \ (\ left \ {\ begin {array } {l} 6x − 2y = 12 \\ 3x + 7y = −13 \ end {array} \ right. \)

Ответ

ⓐ Поскольку одно уравнение уже решено относительно y , использование подстановки будет наиболее удобным.Ⓑ Поскольку оба уравнения имеют стандартную форму, использование исключения будет наиболее удобным.


College Algebra
Учебник 49: Решение систем
Линейных уравнений с двумя переменными


Цели обучения



После изучения этого руководства вы сможете:
  1. Знайте, является ли упорядоченная пара решением системы линейные уравнения в две переменные или нет.
  2. Решите систему линейных уравнений с двумя переменными. путем построения графиков.
  3. Решите систему линейных уравнений с двумя переменными. заменой метод.
  4. Решите систему линейных уравнений с двумя переменными. устранением методом сложения.

Введение



В этом уроке мы будем специально рассматривать системы, которые имеют два уравнения и две неизвестные.Урок 50: Решение систем Линейный Уравнения в трех переменных будут охватывать системы, которые имеют три уравнения и три неизвестных. Мы рассмотрим их решение трех разных способы: путем построения графиков, методом подстановки и путем исключения добавление метод. Итак, давайте посмотрим на эти системы.

Учебник



Система линейных уравнений

Система линейных уравнений состоит из двух или более линейных уравнения, которые решаются одновременно.

В этом руководстве мы рассмотрим системы, имеют только два линейных уравнения и две неизвестные.




В общем, решение системы двух переменных заказанный пара, которая делает ОБЕИХ уравнениями верными.

Другими словами, здесь пересекаются два графика, что у них есть в общем.Итак, если упорядоченная пара является решением одного уравнения, но не другой, то это НЕ решение системы.

Согласованная система — это система, в которой хотя бы одно решение.

Несогласованная система — это система, имеющая нет решения .

Уравнения системы зависимы , если ВСЕ решения одного уравнения являются решениями другого уравнения.В Другие словами, они оказываются той же строкой .

Уравнения системы независимы , если они не разделяют ВСЕ решения . У них может быть одна общая черта, только не все их.




Одно решение
Если система с двумя переменными имеет одно решение, это заказанный пара, которая является решением ОБЕИХ уравнений. Другими словами, когда вы вставляете значения упорядоченной пары, она делает ОБА уравнения ПРАВДА.

Если у вас есть одно решение для окончательного ответа, — это эта система непротиворечива или непоследовательна?
Если вы сказали «последовательный», похлопайте себя по плечу!

Если вы получите одно решение для окончательного ответа, будет уравнения быть зависимыми или независимыми?
Если вы сказали независимый, вы правы!

На приведенном ниже графике показана система двух уравнений. и два неизвестных у которого есть одно решение:


Нет решения
Если две линии параллельны друг другу, они будут никогда не пересекаются. Значит, у них нет ничего общего. В этом ситуация у вас не будет решения.

Если вы не получите окончательного ответа, — это эта система непротиворечива или непоследовательна?
Если вы сказали «непоследовательно», вы правы!

Если вы не получите окончательного ответа, служба уравнения быть зависимыми или независимыми?
Если вы сказали независимый, вы правы!

На приведенном ниже графике показана система двух уравнений. и два неизвестных не имеющий решения:


Бесконечный Решения
Если две линии в конечном итоге лежат друг на друге, тогда Там есть бесконечное количество решений. В этой ситуации они было бы в конечном итоге будут одной и той же строкой, поэтому любое решение, которое будет работать в одном уравнение будет работать в другом.

Если вы получите бесконечное количество решений для Ваш окончательный ответ, я s эта система непротиворечива или непоследовательна?
Если вы сказали «последовательный», вы правы!

Если вы получите бесконечное количество решений для ваш окончательный ответ, будет уравнения быть зависимыми или независимыми?
Если вы сказали иждивенец, вы правы!

График ниже иллюстрирует систему двух уравнений. и два неизвестных имеющий бесконечное количество решений:




Пример 1 : Определите, является ли каждая упорядоченная пара решением из система.
(3, 1) и (0, -1)

Давайте проверим заказанную пару (3, 1) в первом уравнение:



* Вставка 3 для x и 1 для y

* Истинное утверждение


Пока все хорошо, (3, 1) — решение первого уравнение 2 x -3 y = 3.

Теперь давайте проверим (3, 1) во втором уравнении:



* Вставка 3 для x и 1 для y

* Истинное утверждение


Эй, мы закончили с еще одним верным утверждением, которое означает, что (3, 1) есть также решение второго уравнения 4 x — 2 y = 10.

Вот большой вопрос, является ли (3, 1) решением данная система ?????

Поскольку это было решение ОБЕИХ уравнений в система, тогда это это решение для всей системы.

Теперь поместим (0, -1) в первое уравнение:



* Вставка 0 для x и -1 для y

* Истинное утверждение


Это истинное утверждение, поэтому (0, -1) является решением первое уравнение 2 x — 3 y = 3.

Наконец, поместим (0, -1) во второе уравнение:



* Вставка 0 для x и -1 для y

* Ложная выписка


На этот раз мы получили ложное заявление, вы знаете, что это средства. (0, -1) НЕ является решением второго уравнения 4 x — 2 y = 10.

Вот большой вопрос, является ли (0, -1) решением данная система ?????

Так как это не было решением ОБЕИХ уравнений в система, то это не решение всей системы.

Три способа
Решение систем линейных
Уравнений с двумя переменными




Шаг 1: Изобразите первое уравнение.


Если в инструкциях указано иное, вы можете использовать любой «законный» способ построить линию. Если вам нужен обзор линий графика, не стесняйтесь вернуться к Урок 27: Графические линии.

Шаг 2: Изобразите второе уравнение на та же координата система как первая.


Вы изобразите второе уравнение так же, как и любое другое. уравнение. Обратитесь к первому шагу, если вам нужно узнать, как график линия.

Отличие в том, что на такой же ставишь система координат как первый. Это как две задачи с графиком в одной.


Шаг 3: Найдите решение.


Если две линии пересекаются в одном месте , то точка перекресток — решение системы.

Если у две линии параллельны , то они никогда не пересекаются, так что нет решения.

Если у две линии лежат друг над другом , то они та же строка , и у вас есть бесконечное количество решений . В этом случае вы можете записать любое уравнение как решение указывать это одна и та же линия.


Шаг 4: Проверьте предложенный заказанный парное решение в ОБА уравнения.


Предлагаемое решение можно подключить к ОБА уравнения.Если он делает ОБЕИЕ уравнения истинными, тогда у вас есть решение система.

Если хотя бы одно из них становится ложным, вам нужно идти назад и повторить проблема.



Пример 2 : Решите систему уравнений, построив график.




* Вставка 0 для y для x -int
* x -intercept


Перехват по оси x равен (3, 0).

y-перехват



* Вставка 0 для x для y -int
* y -intercept


Перехват по оси Y равен (0, 3).

Найдите другого решение, положив x = 1.



* Вставка 1 для x

Другое решение — (1, 2).

Решения:

х y (х, у)
3 0 (3, 0)
0 3 (0, 3)
1 2 (1, 2)

Построение упорядоченных парных решений и построение линия:






* Вставка 0 для y для x -int
* x -intercept


Перехват по оси x равен (1, 0).

y-перехват



* Вставка 0 для x для y -int

* Инверсная по отношению к мульт. на -1 — это div. по -1

* y -перехват


Перехват по оси Y равен (0, -1).

Найдите другого решение, положив x = 2.



* Вставьте 2 для x
* Сумма 2 обратная. 2

* Инверсная по отношению к мульт. на -1 это div по -1


Другое решение — (2, 1).

Решения:


х y (х, у)
1 0 (1, 0)
0 -1 (0, -1)
2 1 (2, 1)

Построение упорядоченных парных решений и построение линия:



Нам нужно спросить себя, есть ли место, где две линии пересекаются, и если да, то где?

Ответ — да, они пересекаются в (2, 1).



Вы обнаружите, что если вы подключите заказанную пару (2, 1) в ОБЕИХ уравнения исходной системы, что это решение ОБЕИХ из них.

Решение этой системы — (2, 1).




Пример 3 : Решите систему уравнений, построив график.




* Вставка 0 для y для x -int
* x -intercept


Перехват по оси x равен (5, 0).

y-перехват



* Вставка 0 для x для y -int

* y -перехват


Перехват по оси Y равен (0, 5).

Найдите другого решение, положив x = 1.



* Вставить 1 для x
* Прибавление 1 является вспомогательным. 1


Другое решение — (1, 4).

Решения:


х y (х, у)
5 0 (5, 0)
0 5 (0, 5)
1 4 (1, 4)

Построение упорядоченных парных решений и построение линия:





* Вставить 0 для y для x -int
* Сумма, обратная сумме 3, является вспомогательной.3

* Инверсная по отношению к мульт. на -1 — это div. по -1

* x — перехват


Перехват по оси x равен (3, 0).

y-перехват



* Вставка 0 для x для y -int
* y -intercept


Перехват по оси Y равен (0, 3).

Найдите другого решение, положив x = 1.



* Вставка 1 для x


Другое решение — (1, 2).

Решения:


х y (х, у)
3 0 (3, 0)
0 3 (0, 3)
1 2 (1, 2)

Построение упорядоченных парных решений и построение линия:



Нам нужно спросить себя, есть ли место, где две линии пересекаются, и если да, то где?

Ответ — нет, они не пересекаются.Мы иметь два параллельных линий.



Нет заказанных пар для проверки.

Ответа нет.


Решить методом подстановки

Шаг 1. При необходимости упростите.


Это может включать в себя такие вещи, как удаление () и удаление фракций.

Чтобы удалить (): просто используйте свойство distributive.

Чтобы удалить дроби: поскольку дроби — это еще один способ написать деление, а обратное деление — умножение, дробь удаляется на умножение обе стороны ЖК-дисплеем всех ваших фракций.


Шаг 2: Решите один уравнение для любой переменной.


Неважно, какое уравнение вы используете или какое переменная, которую вы выбираете решить для.

Вы хотите сделать это как можно проще. Если один уравнений уже решено для одной из переменных, это быстро и легко способ идти.

Если вам нужно найти переменную, попробуйте выбрать тот, у которого есть 1 как коэффициент. Таким образом, когда вы идете решать это, вы не будет делить на число и рисковать работать с доля (фу !!).


Шаг 3: Заменить что вы попадаете на шаг 2 в другое уравнение.


Вот почему он называется методом подстановки. Убедись в том, что вы подставляете выражение в ДРУГОЕ уравнение, то, которое вы не сделал используйте на шаге 2.

Это даст вам одно уравнение с одним неизвестным.


Шаг 4. Решить для оставшаяся переменная.


Решите уравнение, заданное на шаге 3 для переменной что осталось.

Если вам нужен обзор решения линейных уравнений, прочувствуйте бесплатно перейти к Учебное пособие 14: Линейные уравнения от переменной.

Если ваша переменная выпадает и вы получаете ЛОЖЬ заявление, что означает ваш ответ не решение.

Если ваша переменная выпадает и у вас есть ИСТИНА заявление, что означает ваш ответ — бесконечные решения, которые были бы уравнением линия.


Шаг 5. Решить для вторая переменная.


Если вы нашли значение переменной на шаге 4, что означает два уравнения имеют одно решение. Вставьте значение, найденное в шаг 4 в любое из уравнений задачи и решить для другого Переменная.


Шаг 6. Проверьте предлагаемые упорядоченное парное решение в ОБЕИХ исходных уравнениях.


Предлагаемое решение можно подключить к ОБА уравнения.Если это делает ОБЕИЕ уравнения истинными, тогда у вас есть решение система.

Если хотя бы одно из них становится ложным, вам нужно идти назад и повторить проблема.




Пример 4 : Решите систему уравнений заменой метод:


Оба эти уравнения уже упрощены. Нет необходимости в работе делать здесь.



Неважно, какое уравнение или какую переменную вы используете. выбрать решение для. Просто будьте проще.

Так как x во втором уравнение имеет коэффициент 1, это означало бы, что нам не нужно было бы делить на номер решить эту проблему и рискнуть работать с дробями (YUCK). Самый простой способ — решить второе уравнение относительно x , и мы определенно хотим выбрать легкий путь.

Вы не ошибетесь, если выберете другой уравнение и / или решить для y , вы снова хотите сохранить его как просто насколько возможно.

Решая второе уравнение для x , мы получить:


* 2-е уравнение решено для x



Подставьте выражение y + 1 вместо x в первое уравнение и решите относительно y :
(когда вы вставляете подобное выражение, это похоже на то, как вы подключаете в число вашей переменной)



* Под. л + 1 дюйм для x
* Расст. С 3 по ()
* Объединить похожие термины

* Сумма, обратная сумме 3, является вспомогательной. 3



Вставить 3 для y в уравнение в шаг 2, чтобы найти значение x .


* Вставка 3 для y



Вы обнаружите, что если вы подключите заказанную пару (4, 3) в ОБЕИХ уравнения исходной системы, что это решение ОБЕИХ из них.

(4, 3) — это решение нашей системы.




Пример 5 : Решите систему уравнений заменой метод:


В этом уравнении полно неприятных дробей. Мы можем упростить оба уравнения, умножив каждое в отдельности на его ЖК-дисплей, как вы можете сделать это, когда работаете с одним уравнением.До тех пор, как вы проделайте то же самое с обеими сторонами уравнения, оставив обе стороны равны друг другу.

Умножая каждое уравнение на соответствующий ЖК-дисплей, мы получить:


* Мног. по ЖК-дисплею 2

* Мног. по ЖК-дисплею 2




Обратите внимание, что второе уравнение уже решено для y . Мы можем использовать его на этом этапе.

Неважно, какое уравнение или переменную вы выбрать решение для. Но в ваших интересах, чтобы это было так просто, как возможный.

Второе уравнение, решенное относительно y :


* Решено для y



Подставим выражение -3 x + 4 для y в первое уравнение и решите относительно x :
(когда вы вставляете подобное выражение, это похоже на то, как вы подключаете в число вашей переменной)



* Под.-3 х + 4 для y
* Переменная выпала И ложь


Погодите, а где наш переменная go ????

Как упоминалось выше, если ваша переменная выпадает, и вы иметь оператор FALSE, тогда решения нет. Если бы мы изобразили эти два графика, они будут параллельны друг другу.



Поскольку мы не получили значение для x , там здесь нечего подключать.



Нет заказанных пар для проверки.

Ответа нет.




Пример 6 : Решите систему уравнений заменой метод:


Оба эти уравнения уже упрощены. Нет необходимости в работе делать здесь.



Обратите внимание, что второе уравнение уже решено для y . Мы можем использовать его на этом этапе.

Неважно, какое уравнение или переменную вы выбрать решение для. Но в ваших интересах, чтобы это было так просто, как возможный.

Второе уравнение, решенное относительно y :


* Решено для y



Подставим выражение 2 x — 4 для y в первое уравнение и решите относительно x :
(когда вы вставляете подобное выражение, это похоже на то, как вы подключаете в число вашей переменной)



* Под.2 x — 4 для y

* Переменная выпала И истинно


Погодите, а где наш переменная go ????

Как упоминалось выше, если переменная выпадает И мы иметь ИСТИННОЕ заявление, тогда когда есть бесконечное количество решений.Они в конечном итоге та же линия.



Поскольку мы не получили значение для x , там здесь нечего подключать.



Здесь нет ценности для подключения.

Когда они оказываются в одном уравнении, у вас есть бесконечное число решений.Вы можете написать свой ответ, написав либо уравнение, чтобы указать, что это одно и то же уравнение.

Два способа написать ответ: {( x , y ) | 2 x y = 4} OR {( x , y ) | y = 2 x — 4}.


Решить устранением Метод сложения


Шаг 1. Упростите и при необходимости запишите оба уравнения в виде A x + B y = C.


Это может включать в себя такие вещи, как удаление () и удаление фракций.

Чтобы удалить (): просто используйте свойство distributive.

Чтобы удалить дроби: поскольку дроби — это еще один способ написать деление, а обратное деление — умножение, дробь удаляется на умножение обе стороны ЖК-дисплеем всех ваших фракций.


Шаг 2: Умножьте единицу или оба уравнения на число, которое создаст противоположные коэффициенты для либо x , либо y , если необходимо.


Забегая вперед, мы добавим эти два уравнения вместе . В этом процессе нам нужно убедиться, что одна из переменных падает вне, оставив нам одно уравнение и одно неизвестное.Единственный способ, которым мы можем гарантия, что если мы добавляем противоположности . Сумма противоположности равно 0.

Если ни одна из переменных не выпадает, то мы застреваем с уравнение с две неизвестные, которые неразрешимы.

Неважно, какую переменную вы выберете для удаления вне. Вы хотите, чтобы это было как можно проще.Если переменная уже имеет противоположные коэффициенты, чем при добавлении двух уравнений все вместе. В противном случае вам нужно умножить одно или оба уравнения на число. что создаст противоположные коэффициенты в одной из ваших переменных. Ты может думайте об этом как о ЖК-дисплее. Подумайте, какой номер оригинал коэффициенты оба входят и соответственно умножают каждое отдельное уравнение. Делать убедитесь, что одна переменная положительна, а другая отрицательна, прежде чем вы Добавлять.

Например, если у вас есть 2 x в одном уравнении и 3 x в другом уравнении, мы могли бы умножать первое уравнение на 3 и получаем 6 x и в второе уравнение на -2, чтобы получить -6 x . Так когда вы собираетесь сложить эти два вместе, они выпадут.



Сложите два уравнения.

Переменная с противоположными коэффициентами будет выпадать из этого шаг, и у вас останется одно уравнение с одним неизвестным.


Шаг 4: Решите оставшиеся Переменная.


Решите уравнение, найденное на шаге 3 для переменной что осталось.

Если вам нужен обзор решения линейных уравнений, прочувствуйте бесплатно перейти к Учебное пособие 14: Линейные уравнения от переменной.

Если выпадают обе переменные и вы получаете ЛОЖЬ заявление, что означает ваш ответ не решение.

Если выпадают обе переменные и у вас есть ИСТИНА заявление, что означает ваш ответ — бесконечные решения, которые были бы уравнением линия.


Шаг 5: Решить на секунду Переменная.


Если вы нашли значение переменной на шаге 4, что означает два уравнения имеют одно решение. Вставьте значение, найденное в шаг 4 в любое из уравнений задачи и решить для другого Переменная.


Шаг 6: Проверка предлагаемый упорядоченное парное решение в ОБЕИХ исходных уравнениях.


Предлагаемое решение можно подключить к ОБА уравнения.Если это делает ОБЕИХ уравнения истинными, тогда у вас есть решение система.

Если хотя бы одно из них становится ложным, вам нужно идти назад и повторить проблема.




Пример 7 : Решите систему уравнений с помощью устранение метод:



В этом уравнении полно неприятных дробей. Мы можем упростить оба уравнения, умножив каждое в отдельности на его ЖК-дисплей, как вы можете сделать это, когда работаете с одним уравнением. До тех пор, как вы проделайте то же самое с обеими сторонами уравнения, оставив обе стороны равны друг другу.

Умножая каждое уравнение на соответствующий ЖК-дисплей, мы получить:



* Мног.по ЖК № 6

* Мног. по ЖК 40




Опять же, вы хотите сделать это так же просто, как возможный.

Обратите внимание, как коэффициент при y в первом уравнение равно 2, а во втором уравнении — 5.Нам нужно иметь противоположности, поэтому, если одному из них будет 10, а другому -10, они бы отменить друг друга, когда мы собираемся их добавить. Если бы мы сложили их вместе в том виде, в каком они сейчас, мы бы получили одно уравнение и два переменные, ничего не выпадало. И мы бы не смогли ее решить.

Итак, я предложил умножить первое уравнение на 5 и второй уравнение на -2, , это создаст 10 и -10 перед y ’s и у нас будут свои противоположности.

Умножение первого уравнения на 5 и второго уравнение на -2 получаем:


* Мног. обе стороны 1-го ур. по 5
* Мульт. обе стороны 2-го ур. по -2

* y х иметь противоположное коэффициенты




* Обратите внимание, что y ‘s выпал






Вы можете выбрать любое уравнение, используемое в этой задаче, чтобы вставьте найденное значение x .

Я выбираю 10 для x в первое упрощенное уравнение (найдено на шаге 1), чтобы найти y ’s значение.



* Вставка 10 для x

* Сумма, обратная сумме 30, является вспомогательной. 30

* Инверсная по отношению к мульт.на 2 — div. по 2



Вы обнаружите, что если вы подключите заказанную пару (10, 24) в ОБЕИХ уравнения исходной системы, что это решение ОБЕИХ из их.

(10, 24) — это решение нашей системы.




Пример 8 : Решите систему уравнений с помощью устранение метод:



Эта задача уже упрощена, однако вторая уравнение не записывается в форме A x + B y = C.Другими словами, нам нужно записать его в таком виде, чтобы все выстраивается в линию, когда мы складываем два уравнения вместе.

Переписывая второе уравнение, получаем:



* Инверсия сложения x — sub. x

* Все выстроено



Обратите внимание, что x ‘s коэффициенты уже противоположности.Коэффициенты и равны также противоположности в этом отношении.

Таким образом, нам не нужно умножать любое уравнение на номер.




* Обратите внимание, что x ‘s и y оба выпали



Эй, откуда у нас переменные идти??

Как упоминалось выше, если переменная выпадает И мы иметь ИСТИННОЕ заявление, тогда когда есть бесконечное количество решений.Они в конечном итоге та же линия.



Здесь нет ценности для подключения.



Здесь нет ценности для подключения.

Когда они оказываются в одном уравнении, у вас есть бесконечное число решений.Вы можете написать свой ответ, написав либо уравнение, чтобы указать, что это одно и то же уравнение.

Два способа написать ответ: {( x , y ) | x y = 3} OR {( x , y ) | y = x — 3}.




Пример 9 : Решите систему уравнений с помощью устранение метод:



Оба эти уравнения уже упрощены и в правильная форма. Здесь не нужно ничего делать.



Опять же, вы хотите сделать это так же просто, как возможный.

Обратите внимание, как коэффициент при x в первом уравнение равно 5, а во втором уравнении — 10. Нам нужно имеют противоположности, поэтому, если один из них -10, а другой 10, они бы отменить друг друга, когда мы собираемся их добавить.Если бы мы сложили их вместе в том виде, в каком они сейчас, мы бы получили одно уравнение и два переменные, ничего не выпадало. И мы бы не смогли ее решить.

Итак, я предложил умножить первое уравнение на -2, это создаст -10 и 10 перед x и у нас будут свои противоположности.

Умножая первое уравнение на -2, получаем:


* Мног.обе стороны 1-го ур. по -2

* x ‘s и и имеют противоположные коэффициенты




* Обратите внимание, что x ‘s и y оба выпали



Погодите, а где наш переменная go ????

Как упоминалось выше, если ваша переменная выпадает, и вы иметь оператор FALSE, тогда решения нет. Если бы мы изобразили эти два графика, они будут параллельны друг другу.



Здесь нет ценности для подключения.



Здесь нет ценности для подключения.

Ответ — нет решения.



Практические задачи



Это практические задачи, которые помогут вам перейти на следующий уровень. Это позволит вам проверить и понять, понимаете ли вы эти типы проблем. Математика работает как и все в противном случае, если вы хотите добиться успеха в этом, вам нужно практиковать это. Даже лучшие спортсмены и музыканты получали помощь и много практиковаться, практиковаться, практиковаться, чтобы стать лучше в своем виде спорта или инструменте. На самом деле не бывает слишком много практики.

Чтобы получить от них максимальную пользу, вам следует решить проблему на свой, а затем проверьте свой ответ, щелкнув ссылку для ответа / обсуждения для этой проблемы .По ссылке вы найдете ответ а также любые шаги, которые позволили найти этот ответ.

Практические задачи 1a — 1c: Решите каждую систему с помощью подмена или исключение методом добавления.

Практическая задача 2a: Решите систему, построив график.

Нужна дополнительная помощь по этим темам?





Последний раз редактировал Ким Сьюард 25 марта 2011 г.
Авторские права на все содержание (C) 2002 — 2011, WTAMU и Kim Seward. Все права защищены.

Проверка решений уравнений с двумя переменными

Результаты обучения

  • Учитывая пару координат, определите, является ли она решением уравнения с двумя переменными

Все уравнения, которые мы решали до сих пор, были уравнениями с одной переменной.Почти в каждом случае, когда мы решали уравнение, мы получали ровно одно решение. Процесс решения уравнения закончился такой фразой, как [латекс] x = 4 [/ latex]. Затем мы проверили решение, подставив его обратно в уравнение.
Вот пример линейного уравнения с одной переменной и его одно решение.

[латекс] \ begin {array} {c} 3x + 5 = 17 \ hfill \\ \\ 3x = 12 \ hfill \\ x = 4 \ hfill \ end {array} [/ latex]
Но уравнения могут иметь больше чем одна переменная. Уравнения с двумя переменными можно записать в общем виде [латекс] Ax + By = C [/ latex].Уравнение такого вида называется линейным уравнением с двумя переменными.

Линейное уравнение

Уравнение вида [латекс] Ax + By = C [/ latex], где [latex] A \ text {и} B [/ latex] не равны нулю, называется линейным уравнением с двумя переменными.

Обратите внимание, что слово «линия» является линейным.
Вот пример линейного уравнения с двумя переменными, [latex] x [/ latex] и [latex] y \ text {:} [/ latex]

[латекс] \ color {red} {A} x + \ color {blue} {B} y = \ color {green} {C} [/ latex]

[латекс] x + \ color {blue} {4} y = \ color {green} {8} [/ latex]

[латекс] \ color {red} {A = 1}, \ color {blue} {B = 4}, \ color {green} {C = 8} [/ latex]

Является ли [латекс] y = -5x + 1 [/ latex] линейным уравнением? Судя по всему, это не форма [латекс] Ax + By = C [/ latex].Но мы могли бы переписать его в таком виде.

[латекс] y = -5x + 1 [/ латекс]
Добавьте [латекс] 5x [/ латекс] с обеих сторон. [латекс] y + 5x = -5x + 1 + 5x [/ латекс]
Упростить. [латекс] y + 5x = 1 [/ латекс]
Используйте свойство коммутативности, чтобы поместить его в [латекс] Ax + By = C [/ latex]. [латекс] \ color {красный} {A} x + \ color {синий} {B} y = C [/ latex]

[латекс] 5x + y = 1 [/ латекс]

Переписав [latex] y = -5x + 1 [/ latex] как [latex] 5x + y = 1 [/ latex], мы можем увидеть, что это линейное уравнение с двумя переменными, потому что оно может быть записано в виде [латекс] Ax + By = C [/ латекс].

Линейные уравнения с двумя переменными имеют бесконечно много решений. Для каждого числа, которое заменяется на [латекс] x [/ latex], существует соответствующее значение [latex] y [/ latex]. Эта пара значений является решением линейного уравнения и представлена ​​упорядоченной парой [латекс] \ left (x, y \ right) [/ latex]. Когда мы подставляем эти значения [latex] x [/ latex] и [latex] y [/ latex] в уравнение, результатом будет истинное утверждение, потому что значение в левой части равно значению в правой части.

Решение линейного уравнения с двумя переменными

Упорядоченная пара [латекс] \ left (x, y \ right) [/ latex] является решением линейного уравнения [латекс] Ax + By = C [/ latex], если уравнение является истинным утверждением, когда [ latex] x \ text {-} [/ latex] и [latex] y \ text {-values} [/ latex] упорядоченной пары подставляются в уравнение.

пример

Определите, какие упорядоченные пары являются решениями уравнения [latex] x + 4y = 8 \ text {:} [/ latex]

1.[латекс] \ влево (0,2 \ вправо) [/ латекс]
2. [латекс] \ влево (2, -4 \ вправо) [/ латекс]
3. [латекс] \ влево (-4,3 \ справа) [/ латекс]

Решение
Подставьте [латекс] x \ text {- и} y \ text {-values} [/ latex] из каждой упорядоченной пары в уравнение и определите, является ли результат истинным утверждением.

1. [латекс] \ левый (0,2 \ правый) [/ латекс] 2. [латекс] \ влево (2, -4 \ вправо) [/ латекс] 3. [латекс] \ влево (-4,3 \ вправо) [/ латекс]
[латекс] x = \ color {синий} {0}, y = \ color {красный} {2} [/ latex]

[латекс] x + 4y = 8 [/ латекс]

[латекс] \ color {синий} {0} +4 \ cdot \ color {красный} {2} \ stackrel {?} {=} 8 [/ latex]

[латекс] 0 + 8 \ stackrel {?} {=} 8 [/ латекс]

[латекс] 8 = 8 \ галочка [/ латекс]

[латекс] x = \ color {синий} {2}, y = \ color {красный} {- 4} [/ latex]

[латекс] x + 4y = 8 [/ латекс]

[латекс] \ color {blue} {2} +4 (\ color {red} {- 4}) \ stackrel {?} {=} 8 [/ latex]

[латекс] 2 + (- 16) \ stackrel {?} {=} 8 [/ латекс]

[латекс] -14 = 8 [/ латекс]

[латекс] x = \ color {синий} {- 4}, y = \ color {красный} {3} [/ latex]

[латекс] x + 4y = 8 [/ латекс]

[латекс] \ color {blue} {- 4} +4 \ cdot \ color {red} {3} \ stackrel {?} {=} 8 [/ latex]

[латекс] -4 + 12 \ stackrel {?} {=} 8 [/ латекс]

[латекс] 8 = 8 \ галочка [/ латекс]

[латекс] \ left (0,2 \ right) [/ latex] — это решение. [латекс] \ left (2, -4 \ right) [/ latex] не является решением. [латекс] \ left (-4,3 \ right) [/ latex] — это решение.

пример

Определите, какие упорядоченные пары являются решениями уравнения. [латекс] y = 5x — 1 \ text {:} [/ latex]

1. [латекс] \ left (0, -1 \ right) [/ latex]
2. [латекс] \ left (1,4 \ right) [/ латекс]
3. [латекс] \ left (-2 , -7 \ вправо) [/ латекс]

Показать решение

Решение
Подставьте [латекс] x \ text {-} [/ latex] и [latex] y \ text {-values} [/ latex] из каждой упорядоченной пары в уравнение и определите, приводит ли оно к истинному утверждению.

1. [латекс] \ слева (0, -1 \ справа) [/ латекс] 2. [латекс] \ левый (1,4 \ правый) [/ латекс] 3. [латекс] \ влево (-2, -7 \ вправо) [/ латекс]
[латекс] x = \ color {blue} {0}, y = \ color {red} {- 1} [/ latex]

[латекс] y = 5x-1 [/ латекс]

[латекс] \ color {red} {- 1} \ stackrel {?} {=} 5 (\ color {blue} {o}) — 1 [/ latex]

[латекс] -1 \ stackrel {?} {=} 0-1 [/ латекс]

[латекс] -1 = -1 \ галочка [/ латекс]

[латекс] x = \ color {синий} {1}, y = \ color {красный} {4} [/ latex]

[латекс] y = 5x-1 [/ латекс]

[латекс] \ color {red} {4} \ stackrel {?} {=} 5 (\ color {blue} {1}) — 1 [/ latex]

[латекс] 4 \ stackrel {?} {=} 5-1 [/ латекс]

[латекс] 4 = 4 \ галочка [/ латекс]

[латекс] x = \ color {синий} {- 2}, y = \ color {красный} {- 7} [/ latex]

[латекс] y = 5x-1 [/ латекс]

[латекс] \ color {red} {- 7} \ stackrel {?} {=} 5 (\ color {blue} {- 2}) — 1 [/ latex]

[латекс] -7 \ stackrel {?} {=} — 10-1 [/ латекс]

[латекс] -7 = -11 [/ латекс]

[latex] \ left (0, -1 \ right) [/ latex] — это решение. [латекс] \ left (1,4 \ right) [/ latex] — это решение. [латекс] \ left (-2, -7 \ right) [/ latex] не является решением.

Системы линейных уравнений: две переменные

Результаты обучения

  • Решайте системы уравнений с помощью построения графиков, подстановок и сложений.
  • Определите несовместимые системы уравнений, содержащие две переменные.
  • Выразите решение системы зависимых уравнений, содержащей две переменные, в стандартных обозначениях.

Производитель скейтбордов представляет новую линейку досок. Производитель отслеживает свои затраты, то есть сумму, которую он тратит на производство плат, и свой доход, который представляет собой сумму, которую он зарабатывает от продажи своих плат. Как компания может определить, получает ли она прибыль от своей новой линии? Сколько скейтбордов необходимо произвести и продать, чтобы можно было получить прибыль? В этом разделе мы рассмотрим линейные уравнения с двумя переменными, чтобы ответить на эти и подобные вопросы.

(предоставлено Thomas Sørenes)

Введение в системные решения

Чтобы исследовать такие ситуации, как ситуация с производителем скейтборда, нам необходимо признать, что мы имеем дело с более чем одной переменной и, вероятно, более чем с одним уравнением. Система линейных уравнений состоит из двух или более линейных уравнений, составленных из двух или более переменных, так что все уравнения в системе рассматриваются одновременно. Чтобы найти единственное решение системы линейных уравнений, мы должны найти числовое значение для каждой переменной в системе, которое будет удовлетворять всем уравнениям в системе одновременно.Некоторые линейные системы могут не иметь решения, а другие могут иметь бесконечное количество решений. Чтобы линейная система имела единственное решение, должно быть по крайней мере столько же уравнений, сколько переменных. Даже в этом случае это не гарантирует уникального решения.

В этом разделе мы рассмотрим системы линейных уравнений с двумя переменными, которые состоят из двух уравнений, содержащих две разные переменные. Например, рассмотрим следующую систему линейных уравнений с двумя переменными.

[латекс] \ begin {align} 2x + y & = 15 \\ [1 мм] 3x-y & = 5 \ end {align} [/ latex]

Решение системы линейных уравнений с двумя переменными — это любая упорядоченная пара, которая удовлетворяет каждому уравнению независимо. В этом примере упорядоченная пара [латекс] (4,7) [/ латекс] является решением системы линейных уравнений. Мы можем проверить решение, подставив значения в каждое уравнение, чтобы увидеть, удовлетворяет ли упорядоченная пара обоим уравнениям. Вскоре мы исследуем способы нахождения такого решения, если оно существует.

[латекс] \ begin {align} 2 \ left (4 \ right) + \ left (7 \ right) & = 15 && \ text {True} \\ [1 мм] 3 \ left (4 \ right) — \ left (7 \ right) & = 5 && \ text {True} \ end {align} [/ latex]

Помимо учета количества уравнений и переменных, мы можем классифицировать системы линейных уравнений по количеству решений. Согласованная система уравнений имеет по крайней мере одно решение. Согласованной системой считается независимая система , если она имеет единственное решение, такое как пример, который мы только что исследовали.Две линии имеют разные уклоны и пересекаются в одной точке на плоскости. Согласованной системой считается зависимая система , если уравнения имеют одинаковый наклон и одинаковые точки пересечения y . Другими словами, линии совпадают, поэтому уравнения представляют одну и ту же линию. Каждая точка на линии представляет пару координат, удовлетворяющую системе. Таким образом, существует бесконечное количество решений.

Другой тип системы линейных уравнений — это несовместимая система , в которой уравнения представляют собой две параллельные линии.Линии имеют одинаковый наклон и разные точки пересечения y- . Для обеих линий нет общих точек; следовательно, у системы нет решения.

Общее примечание: типы линейных систем

Существует три типа систем линейных уравнений с двумя переменными и три типа решений.

  • Независимая система имеет ровно одну пару решений [latex] \ left (x, y \ right) [/ latex]. Точка пересечения двух линий — единственное решение.
  • Несогласованная система не имеет решения. Обратите внимание, что две линии параллельны и никогда не пересекутся.
  • Зависимая система имеет бесконечно много решений. Линии совпадают. Это одна и та же линия, поэтому каждая пара координат на линии является решением обоих уравнений.

Ниже приводится сравнение графических представлений каждого типа системы.

Практическое руководство. Для данной системы линейных уравнений и упорядоченной пары определите, является ли упорядоченная пара решением.

  1. Подставьте упорядоченную пару в каждое уравнение системы.
  2. Определите, являются ли истинные утверждения результатом подстановки в обоих уравнениях; в таком случае заказанная пара является решением.

Пример: определение того, является ли упорядоченная пара решением системы уравнений

Определите, является ли упорядоченная пара [латекс] \ left (5,1 \ right) [/ latex] решением данной системы уравнений.

[латекс] \ begin {align} x + 3y & = 8 \\ 2x-9 & = y \ end {align} [/ latex]

Показать решение

Подставьте упорядоченную пару [латекс] \ left (5,1 \ right) [/ latex] в оба уравнения.

[латекс] \ begin {align} \ left (5 \ right) +3 \ left (1 \ right) & = 8 \\ [1mm] 8 & = 8 && \ text {True} \\ [3mm] 2 \ left (5 \ right) -9 & = \ left (1 \ right) \\ [1 мм] 1 & = 1 && \ text {True} \ end {align} [/ latex]

Упорядоченная пара [латекс] \ left (5,1 \ right) [/ latex] удовлетворяет обоим уравнениям, поэтому это решение системы.

Анализ решения

Мы можем ясно увидеть решение, построив график каждого уравнения. Поскольку решение представляет собой упорядоченную пару, удовлетворяющую обоим уравнениям, это точка на обеих прямых и, следовательно, точка пересечения двух прямых.

Попробуй

Определите, является ли упорядоченная пара [латекс] \ left (8,5 \ right) [/ latex] решением следующей системы.

[латекс] \ begin {align} 5x-4y & = 20 \\ 2x + 1 & = 3y \ end {align} [/ latex]

Решение систем уравнений с помощью построения графиков

Существует несколько методов решения систем линейных уравнений. Для системы линейных уравнений с двумя переменными мы можем определить как тип системы, так и решение, построив систему уравнений на одном и том же наборе осей.

Пример: решение системы уравнений с двумя переменными с помощью построения графиков

Решите следующую систему уравнений, построив график. Определите тип системы.

[латекс] \ begin {align} 2x + y & = — 8 \\ x-y & = — 1 \ end {align} [/ latex]

Показать решение

Решите первое уравнение для [латекс] y [/ латекс].

[латекс] \ begin {align} 2x + y & = — 8 \\ y & = — 2x-8 \ end {align} [/ latex]

Решите второе уравнение для [латекс] y [/ латекс].

[латекс] \ begin {align} x-y & = — 1 \\ y & = x + 1 \ end {align} [/ latex]

Изобразите оба уравнения на одном и том же наборе осей:

Кажется, что линии пересекаются в точке [латекс] \ влево (-3, -2 \ вправо) [/ латекс].Мы можем убедиться, что это решение системы, подставив упорядоченную пару в оба уравнения.

[латекс] \ begin {align} 2 \ left (-3 \ right) + \ left (-2 \ right) & = — 8 \\ [1 мм] -8 = -8 && \ text {True} \\ [ 3 мм] \ left (-3 \ right) — \ left (-2 \ right) & = — 1 \\ [1 мм] -1 & = — 1 && \ text {True} \ end {align} [/ latex]

Решением системы является упорядоченная пара [латекс] \ left (-3, -2 \ right) [/ latex], поэтому система независима.

Попробуй

Решите следующую систему уравнений, построив график.

[латекс] \ begin {собрано} 2x — 5y = -25 \\ -4x + 5y = 35 \ end {собрано} [/ latex]

Показать решение

Решением системы является упорядоченная пара [латекс] \ left (-5,3 \ right) [/ latex].

Вопросы и ответы

Можно ли использовать построение графиков, если система непоследовательна или зависима?

Да, в обоих случаях мы все еще можем построить график системы для определения типа системы и решения. Если две линии параллельны, система не имеет решения и непоследовательна.Если две линии идентичны, система имеет бесконечное количество решений и является зависимой системой.

Попробуй

Постройте три разные системы с помощью онлайн-графического инструмента. Отнесите каждое решение к категории непротиворечивых или непоследовательных. Если система согласована, определите, является ли она зависимой или независимой. Возможно, вам будет проще построить график каждой системы по отдельности, а затем очистить свои записи, прежде чем строить следующую.
1)
[латекс] 5x-3y = -19 [/ латекс]
[латекс] x = 2y-1 [/ латекс]

2)
[латекс] 4x + y = 11 [/ латекс]
[латекс] -2y = -25 + 8x [/ latex]

3)
[латекс] y = -3x + 6 [/ latex]
[латекс] — \ frac {1} {3} y + 2 = x [/ latex]

Показать решение
  1. Одно решение — последовательное, независимое
  2. Нет решений, непоследовательные, ни зависимые, ни независимые
  3. Множество решений — последовательные, зависимые

Решение систем уравнений подстановкой

Решение линейной системы двух переменных с помощью построения графиков хорошо работает, когда решение состоит из целых значений, но если наше решение содержит десятичные дроби или дроби, это не самый точный метод.Мы рассмотрим еще два метода решения системы линейных уравнений , которые более точны, чем построение графиков. Одним из таких методов является решение системы уравнений методом подстановки , в котором мы решаем одно из уравнений для одной переменной, а затем подставляем результат во второе уравнение, чтобы найти вторую переменную. Напомним, что мы можем решать только одну переменную за раз, поэтому метод подстановки является одновременно ценным и практичным.

Как: дана система двух уравнений с двумя переменными, решите, используя метод подстановки.

  1. Решите одно из двух уравнений относительно одной из переменных через другую.
  2. Подставьте выражение для этой переменной во второе уравнение, затем найдите оставшуюся переменную.
  3. Подставьте это решение в любое из исходных уравнений, чтобы найти значение первой переменной. Если возможно, запишите решение в виде упорядоченной пары.
  4. Проверьте решение в обоих уравнениях.

Пример: решение системы уравнений с двумя переменными подстановкой

Решите следующую систему уравнений путем подстановки.

[латекс] \ begin {align} -x + y & = — 5 \\ 2x-5y & = 1 \ end {align} [/ latex]

Показать решение

Сначала мы решим первое уравнение для [латекс] y [/ латекс].

[латекс] \ begin {align} -x + y & = — 5 \\ y & = x — 5 \ end {align} [/ latex]

Теперь мы можем заменить выражение [latex] x — 5 [/ latex] на [latex] y [/ latex] во втором уравнении.

[латекс] \ begin {align} 2x — 5y & = 1 \\ 2x — 5 \ left (x — 5 \ right) & = 1 \\ 2x — 5x + 25 & = 1 \\ -3x & = — 24 \\ x & = 8 \ end {align} [/ latex]

Теперь мы подставляем [latex] x = 8 [/ latex] в первое уравнение и решаем относительно [latex] y [/ latex].

[латекс] \ begin {align} — \ left (8 \ right) + y & = — 5 \\ y & = 3 \ end {align} [/ latex]

Наше решение — [латекс] \ left (8,3 \ right) [/ latex].

Проверьте решение, подставив [latex] \ left (8,3 \ right) [/ latex] в оба уравнения.

[латекс] \ begin {align} -x + y & = — 5 \\ — \ left (8 \ right) + \ left (3 \ right) & = — 5 && \ text {True} \\ [3mm] 2x — 5y & = 1 \\ 2 \ left (8 \ right) -5 \ left (3 \ right) & = 1 && \ text {True} \ end {align} [/ latex]

Попробуй

Решите следующую систему уравнений путем подстановки.

[латекс] \ begin {align} x & = y + 3 \\ 4 & = 3x — 2y \ end {align} [/ latex]

Показать решение

[латекс] \ влево (-2, -5 \ вправо) [/ латекс]

Вопросы и ответы

Можно ли методом подстановки решить любую линейную систему с двумя переменными?

Да, но этот метод работает лучше всего, если одно из уравнений содержит коэффициент 1 или –1, чтобы нам не приходилось иметь дело с дробями.

Следующее видео длится ~ 10 минут и представляет собой мини-урок по использованию метода подстановки для решения системы линейных уравнений.Мы представляем три разных примера, а также используем инструмент построения графиков, чтобы подытожить решение для каждого примера.

Решение систем уравнений с двумя переменными методом сложения

Третий метод решения систем линейных уравнений — это метод сложения , этот метод также называется методом исключения . В этом методе мы складываем два члена с одинаковой переменной, но с противоположными коэффициентами, так что сумма равна нулю.Конечно, не все системы созданы с двумя членами одной переменной, имеющими противоположные коэффициенты. Часто нам приходится корректировать одно или оба уравнения путем умножения, чтобы одна переменная была исключена сложением.

Как: решить систему уравнений методом сложения.

  1. Запишите оба уравнения с переменными x и y слева от знака равенства и константами справа.
  2. Напишите одно уравнение над другим, выровняв соответствующие переменные.Если одна из переменных в верхнем уравнении имеет коэффициент, противоположный той же переменной в нижнем уравнении, сложите уравнения вместе, исключив одну переменную. Если нет, используйте умножение на ненулевое число, чтобы одна из переменных в верхнем уравнении имела коэффициент, противоположный той же переменной в нижнем уравнении, затем добавьте уравнения, чтобы исключить переменную.
  3. Решите полученное уравнение для оставшейся переменной.
  4. Подставьте это значение в одно из исходных уравнений и решите для второй переменной.
  5. Проверьте решение, подставив значения в другое уравнение.

Пример: решение системы методом сложения

Решите данную систему уравнений сложением.

[латекс] \ begin {align} x + 2y & = — 1 \\ -x + y & = 3 \ end {align} [/ latex]

Показать решение

Оба уравнения уже установлены равными константе. Обратите внимание, что коэффициент [латекс] x [/ латекс] во втором уравнении, –1, является противоположностью коэффициента [латекс] x [/ латекс] в первом уравнении, 1.Мы можем сложить два уравнения, чтобы исключить [latex] x [/ latex] без умножения на константу.

[латекс] \ begin {align} x + 2y & = — 1 \\ -x + y & = 3 \\ \ hline 3y & = 2 \ end {align} [/ latex]

Теперь, когда мы удалили [latex] x [/ latex], мы можем решить полученное уравнение для [latex] y [/ latex].

[латекс] \ begin {align} 3y & = 2 \\ y & = \ dfrac {2} {3} \ end {align} [/ latex]

Затем мы подставляем это значение для [latex] y [/ latex] в одно из исходных уравнений и решаем для [latex] x [/ latex].

[латекс] \ begin {align} -x + y & = 3 \\ -x + \ frac {2} {3} & = 3 \\ -x & = 3- \ frac {2} {3} \\ -x & = \ frac {7} {3} \\ x & = — \ frac {7} {3} \ end {align} [/ latex]

Решение этой системы — [латекс] \ left (- \ frac {7} {3}, \ frac {2} {3} \ right) [/ latex].

Проверьте решение в первом уравнении.

[латекс] \ begin {align} x + 2y & = — 1 \\ \ left (- \ frac {7} {3} \ right) +2 \ left (\ frac {2} {3} \ right) & = \\ — \ frac {7} {3} + \ frac {4} {3} & = \\ \ — \ frac {3} {3} & = \\ -1 & = — 1 && \ text {True} \ end {align} [/ латекс]

Анализ решения

Мы получаем важное представление о системах уравнений, глядя на графическое представление.Посмотрите на график ниже, чтобы увидеть, что уравнения пересекаются в решении. Нам не нужно спрашивать, может ли быть второе решение, потому что наблюдение за графиком подтверждает, что система имеет ровно одно решение.

Пример: использование метода сложения, когда требуется умножение одного уравнения

Решите данную систему уравнений методом сложения .

[латекс] \ begin {align} 3x + 5y & = — 11 \\ x — 2y & = 11 \ end {align} [/ latex]

Показать решение

Добавление этих уравнений в представленном виде не устраняет переменную.Однако мы видим, что в первом уравнении есть [latex] 3x [/ latex], а во втором уравнении — [latex] x [/ latex]. Итак, если мы умножим второе уравнение на [latex] -3, \ text {} [/ latex], термины x прибавятся к нулю.

[латекс] \ begin {align} x — 2y & = 11 \\ -3 \ left (x — 2y \ right) & = — 3 \ left (11 \ right) && \ text {Умножаем обе стороны на} -3 \ \ -3x + 6y & = — 33 && \ text {Использовать свойство распределения}. \ end {align} [/ latex]

А теперь добавим их.

[латекс] \ begin {align} 3x + 5y & = — 11 \\ −3x + 6y & = — 33 \\ \ hline 11y & = — 44 \\ y & = — 4 \ end {align} [/ latex]

На последнем этапе мы подставляем [latex] y = -4 [/ latex] в одно из исходных уравнений и решаем для [latex] x [/ latex].

[латекс] \ begin {align} 3x + 5y & = — 11 \\ 3x + 5 \ left (-4 \ right) & = — 11 \\ 3x — 20 & = — 11 \\ 3x & = 9 \\ x & = 3 \ end {align} [/ latex]

Наше решение — упорядоченная пара [латекс] \ left (3, -4 \ right) [/ latex]. Проверьте решение в исходном втором уравнении.

[латекс] \ begin {align} x — 2y & = 11 \\ \ left (3 \ right) -2 \ left (-4 \ right) & = 3 + 8 \\ & = 11 && \ text {True} \ конец {align} [/ latex]

Попробуй

Решите систему уравнений сложением.

[латекс] \ begin {align} 2x — 7y & = 2 \\ 3x + y & = — 20 \ end {align} [/ latex]

Показать решение

[латекс] \ влево (-6, -2 \ вправо) [/ латекс]

Пример: использование метода сложения, когда требуется умножение обоих уравнений

Решите данную систему уравнений с двумя переменными сложением.

[латекс] \ begin {align} 2x + 3y & = — 16 \\ 5x — 10y & = 30 \ end {align} [/ latex]

Показать решение

Одно уравнение имеет [латекс] 2x [/ латекс], а другое — [латекс] 5x [/ латекс].Наименьшее общее кратное — [latex] 10x [/ latex], поэтому нам придется умножить оба уравнения на константу, чтобы исключить одну переменную. Давайте удалим [latex] x [/ latex], умножив первое уравнение на [latex] -5 [/ latex], а второе уравнение на [latex] 2 [/ latex].

[латекс] \ begin {align} -5 \ left (2x + 3y \ right) & = — 5 \ left (-16 \ right) \\ -10x — 15y & = 80 \\ [3 мм] 2 \ left (5x — 10y \ right) & = 2 \ left (30 \ right) \\ 10x — 20y & = 60 \ end {align} [/ latex]

Затем мы складываем два уравнения.

[латекс] \ begin {align} -10x-15y & = 80 \\ 10x-20y & = 60 \\ \ hline -35y & = 140 \\ y & = — 4 \ end {align} [/ latex]

Подставьте [латекс] y = -4 [/ latex] в исходное первое уравнение.

[латекс] \ begin {align} 2x + 3 \ left (-4 \ right) & = — 16 \\ 2x — 12 & = — 16 \\ 2x & = — 4 \\ x & = — 2 \ end {align} [ / латекс]

Решение: [латекс] \ left (-2, -4 \ right) [/ latex]. Проверьте это в другом уравнении.

[латекс] \ begin {align} 5x — 10y & = 30 \\ 5 \ left (-2 \ right) -10 \ left (-4 \ right) & = 30 \\ -10 + 40 & = 30 \\ 30 & = 30 \ end {align} [/ latex]

Пример: использование метода сложения в системах уравнений, содержащих дроби

Решите данную систему уравнений с двумя переменными сложением.

[латекс] \ begin {align} \ frac {x} {3} + \ frac {y} {6} & = 3 \\ [1 мм] \ frac {x} {2} — \ frac {y} {4 } & = 1 \ end {align} [/ latex]

Показать решение

Сначала очистите каждое уравнение от дробей, умножив обе части уравнения на наименьший общий знаменатель.

[латекс] \ begin {align} 6 \ left (\ frac {x} {3} + \ frac {y} {6} \ right) & = 6 \ left (3 \ right) \\ [1 мм] 2x + y & = 18 \\ [3 мм] 4 \ left (\ frac {x} {2} — \ frac {y} {4} \ right) & = 4 \ left (1 \ right) \\ [1 мм] 2x-y & = 4 \ end {align} [/ latex]

Теперь умножьте второе уравнение на [latex] -1 [/ latex], чтобы мы могли исключить x .

[латекс] \ begin {align} -1 \ left (2x-y \ right) & = — 1 \ left (4 \ right) \\ [1 мм] -2x + y & = — 4 \ end {align} [/ латекс]

Сложите два уравнения, чтобы исключить x , и решите полученное уравнение относительно y .

[латекс] \ begin {align} 2x + y & = 18 \\ −2x + y & = — 4 \\ \ hline 2y & = 14 \\ y & = 7 \ end {align} [/ latex]

Подставьте [латекс] y = 7 [/ латекс] в первое уравнение.

[латекс] \ begin {align} 2x + \ left (7 \ right) & = 18 \\ 2x & = 11 \\ x & = \ frac {11} {2} \\ & = 7.5 \ end {align} [/ latex]

Решение: [латекс] \ left (\ frac {11} {2}, 7 \ right) [/ latex]. Проверьте это в другом уравнении.

[латекс] \ begin {align} \ frac {x} {2} — \ frac {y} {4} & = 1 \\ [1 мм] \ frac {\ frac {11} {2}} {2} — \ frac {7} {4} & = 1 \\ [1 мм] \ frac {11} {4} — \ frac {7} {4} & = 1 \\ [1 мм] \ frac {4} {4} & = 1 \ end {align} [/ latex]

Попробуй

Решите систему уравнений сложением.

[латекс] \ begin {align} 2x + 3y & = 8 \\ 3x + 5y & = 10 \ end {align} [/ latex]

Показать решение

[латекс] \ влево (10, -4 \ вправо) [/ латекс]

В следующем видео мы представляем больше примеров того, как использовать метод сложения (исключения) для решения системы двух линейных уравнений.

Классифицируйте решения по системам

Теперь, когда у нас есть несколько методов решения систем уравнений, мы можем использовать эти методы для идентификации несовместимых систем. Напомним, что несовместимая система состоит из параллельных линий, которые имеют одинаковый наклон, но разные точки пересечения [latex] y [/ latex]. Они никогда не пересекутся. При поиске решения несовместимой системы мы получим ложное утверждение, например [latex] 12 = 0 [/ latex].

Пример: решение несовместимой системы уравнений

Решите следующую систему уравнений.

[латекс] \ begin {gather} & x = 9 — 2y \\ & x + 2y = 13 \ end {gather} [/ latex]

Показать решение

Мы можем подойти к этой проблеме двумя способами. Поскольку одно уравнение для [латекс] x [/ латекс] уже решено, наиболее очевидным шагом является использование замены.

[латекс] \ begin {align} x + 2y & = 13 \\ \ left (9 — 2y \ right) + 2y & = 13 \\ 9 + 0y & = 13 \\ 9 & = 13 \ end {align} [/ latex]

Ясно, что это утверждение противоречит тому, что [латекс] 9 \ ne 13 [/ латекс].Следовательно, у системы нет решения.

Второй подход заключается в том, чтобы сначала манипулировать уравнениями так, чтобы они оба были в форме пересечения наклона. Мы манипулируем первым уравнением следующим образом.

[латекс] \ begin {собрано} x = 9 — 2y \\ 2y = -x + 9 \\ y = — \ frac {1} {2} x + \ frac {9} {2} \ end {собрано} [ / латекс]

Затем мы преобразуем второе уравнение в форму пересечения наклона.

[латекс] \ begin {собрано} x + 2y = 13 \\ 2y = -x + 13 \\ y = — \ frac {1} {2} x + \ frac {13} {2} \ end {собрано} [ / латекс]

Сравнивая уравнения, мы видим, что они имеют одинаковый наклон, но разные точки пересечения y .Следовательно, линии параллельны и не пересекаются.

[латекс] \ begin {gather} y = — \ frac {1} {2} x + \ frac {9} {2} \\ y = — \ frac {1} {2} x + \ frac {13} {2 } \ end {gather} [/ latex]

Анализ решения

Запись уравнений в форме пересечения наклона подтверждает, что система несовместима, потому что все линии в конечном итоге будут пересекаться, если они не параллельны. Параллельные линии никогда не пересекаются; таким образом, две линии не имеют общих точек. Графики уравнений в этом примере показаны ниже.

Попробуй

Решите следующую систему уравнений с двумя переменными.

[латекс] \ begin {собранный} 2y — 2x = 2 \\ 2y — 2x = 6 \ end {собранный} [/ latex]

Показать решение

Нет решения. Это противоречивая система.

Выражение решения системы зависимых уравнений, содержащих две переменные

Напомним, что зависимая система уравнений с двумя переменными — это система, в которой два уравнения представляют собой одну и ту же линию.Зависимые системы имеют бесконечное количество решений, потому что все точки на одной линии также находятся на другой линии. После использования замены или добавления результирующее уравнение будет идентичным, например [латекс] 0 = 0 [/ латекс].

Пример: поиск решения зависимой системы линейных уравнений

Найдите решение системы уравнений с помощью метода сложения .

[латекс] \ begin {собрано} x + 3y = 2 \\ 3x + 9y = 6 \ end {собрано} [/ latex]

Показать решение

С помощью метода сложения мы хотим исключить одну из переменных, добавив уравнения.В этом случае давайте сосредоточимся на удалении [латекс] х [/ латекс]. Если мы умножим обе части первого уравнения на [latex] -3 [/ latex], то мы сможем исключить переменную [latex] x [/ latex].

[латекс] \ begin {align} x + 3y & = 2 \\ \ left (-3 \ right) \ left (x + 3y \ right) & = \ left (-3 \ right) \ left (2 \ right) \\ -3x — 9y & = — 6 \ end {align} [/ latex]

Теперь сложите уравнения.

[латекс] \ begin {align} −3x − 9y & = — 6 \\ + 3x + 9y & = 6 \\ \ hline 0 & = 0 \ end {align} [/ latex]

Мы видим, что будет бесконечное количество решений, удовлетворяющих обоим уравнениям.

Анализ решения

Если бы мы переписали оба уравнения в форме пересечения наклона, мы могли бы знать, как будет выглядеть решение перед добавлением. Давайте посмотрим, что происходит, когда мы преобразуем систему в форму с пересечением наклона.

[латекс] \ begin {align} \ begin {gather} x + 3y = 2 \\ 3y = -x + 2 \\ y = — \ frac {1} {3} x + \ frac {2} {3} \ конец {собрано} \ hspace {2cm} \ begin {gather} 3x + 9y = 6 \\ 9y = -3x + 6 \\ y = — \ frac {3} {9} x + \ frac {6} {9} \ \ y = — \ frac {1} {3} x + \ frac {2} {3} \ end {gather} \ end {align} [/ latex]

Посмотрите на график ниже.Обратите внимание, что результаты такие же. Общее решение системы — [латекс] \ left (x, — \ frac {1} {3} x + \ frac {2} {3} \ right) [/ latex].

Написание общего решения

В предыдущем примере мы представили анализ решения следующей системы уравнений:

[латекс] \ begin {собрано} x + 3y = 2 \\ 3x + 9y = 6 \ end {собрано} [/ latex]

После небольшой алгебры мы обнаружили, что эти два уравнения в точности совпадают. Затем мы записали общее решение как [latex] \ left (x, — \ frac {1} {3} x + \ frac {2} {3} \ right) [/ latex].Зачем нам писать решение именно так? В некотором смысле это представление о многом говорит нам. Он говорит нам, что x может быть любым, x x . Это также говорит нам, что y будет зависеть от x , точно так же, как когда мы пишем правило функции. В этом случае, в зависимости от того, что вы указали для x , y будет определено в терминах x как [латекс] — \ frac {1} {3} x + \ frac {2} {3} [ /латекс].

Другими словами, существует бесконечно много пар ( x , y ), которые удовлетворяют этой системе уравнений, и все они попадают на линию [латекс] f (x) — \ frac {1} {3} x + \ frac {2} {3} [/ латекс].

Попробуй

Решите следующую систему уравнений с двумя переменными.

[латекс] \ begin {собрано} y — 2x = 5 \\ -3y + 6x = -15 \ end {собрано} [/ latex]

Показать решение

Система является зависимой, поэтому существует бесконечно много решений вида [латекс] \ left (x, 2x + 5 \ right) [/ latex].

Использование систем уравнений для исследования прибыли

Используя то, что мы узнали о системах уравнений, мы можем вернуться к проблеме производства скейтбордов в начале раздела.Функция выручки производителя скейтбордов — это функция, используемая для расчета суммы денег, которая поступает в бизнес. Это может быть представлено уравнением [латекс] R = xp [/ latex], где [latex] x = [/ latex] количество и [latex] p = [/ latex] цена. Функция дохода показана оранжевым цветом на графике ниже.

Функция затрат — это функция, используемая для расчета затрат на ведение бизнеса. Он включает постоянные затраты, такие как аренда и заработная плата, и переменные затраты, такие как коммунальные услуги.Функция стоимости показана синим цветом на графике ниже. Ось x представляет количество в сотнях единиц. Ось y представляет собой стоимость или доход в сотнях долларов.

Точка пересечения двух линий называется точкой безубыточности . Из графика видно, что если произведено 700 единиц, стоимость составит 3300 долларов, а выручка также составит 3300 долларов. Другими словами, компания сломается, даже если произведет и продаст 700 единиц. Они не зарабатывают и не теряют деньги.

Заштрихованная область справа от точки безубыточности представляет объемы, от которых компания получает прибыль. Заштрихованная область слева представляет объемы, по которым компания терпит убытки. Функция прибыли — это функция дохода за вычетом функции затрат, записываемая как [латекс] P \ left (x \ right) = R \ left (x \ right) -C \ left (x \ right) [/ latex]. Очевидно, что знание количества, при котором затраты равны выручке, имеет большое значение для бизнеса.

Пример: определение точки безубыточности и функции прибыли с помощью подстановки

Дана функция стоимости [латекс] C \ left (x \ right) = 0.85x + 35 {,} 000 [/ latex] и функция дохода [latex] R \ left (x \ right) = 1,55x [/ latex], найдите точку безубыточности и функцию прибыли.

Показать решение

Напишите систему уравнений, используя [latex] y [/ latex], чтобы заменить обозначение функции.

[латекс] \ begin {align} y & = 0,85x + 35 {,} 000 \\ y & = 1,55x \ end {align} [/ latex]

Подставьте выражение [latex] 0.85x + 35 {,} 000 [/ latex] из первого уравнения во второе уравнение и решите относительно [latex] x [/ latex].

[латекс] \ begin {собрано} 0.85x + 35 {,} 000 = 1,55x \\ 35 {,} 000 = 0,7x \\ 50 {,} 000 = x \ end {в собранном виде} [/ latex]

Затем мы подставляем [латекс] x = 50 {,} 000 [/ latex] либо в функцию стоимости, либо в функцию дохода.

[латекс] 1,55 \ слева (50 {,} 000 \ справа) = 77 {,} 500 [/ латекс]

Точка безубыточности — [латекс] \ left (50 {,} 000,77 {,} 500 \ right) [/ latex].

Функция прибыли находится по формуле [латекс] P \ left (x \ right) = R \ left (x \ right) -C \ left (x \ right) [/ latex].

[латекс] \ begin {align} P \ left (x \ right) & = 1.55x- \ left (0,85x + 35 {,} 000 \ right) \\ & = 0,7x — 35 {,} 000 \ end {align} [/ latex]

Функция прибыли [латекс] P \ left (x \ right) = 0,7x — 35 {,} 000 [/ latex].

Анализ решения

Стоимость производства 50 000 единиц составляет 77 500 долларов США, а выручка от продажи 50 000 единиц также составляет 77 500 долларов США. Чтобы получить прибыль, бизнес должен произвести и продать более 50 000 единиц.

Из графика ниже видно, что функция прибыли имеет отрицательное значение до тех пор, пока [latex] x = 50 {,} 000 [/ latex] не пересечет ось x .Затем график переходит в положительные значения y и продолжает движение по этому пути, поскольку функция прибыли представляет собой прямую линию. Это показывает, что точка безубыточности для предприятий наступает, когда функция прибыли равна 0. Область слева от точки безубыточности представляет работу с убытками.

Написание системы линейных уравнений для ситуации

Редко можно получить уравнения, которые точно моделируют поведение, с которым вы сталкиваетесь в бизнесе, скорее, вы, скорее всего, столкнетесь с ситуацией, для которой вы знаете ключевую информацию, как в приведенном выше примере.Ниже мы суммируем три ключевых фактора, которые помогут вам преобразовать ситуацию в систему.

Как сделать: в ситуации, которая представляет собой систему линейных уравнений, напишите систему уравнений и найдите решение.

  1. Определите входные и выходные данные каждой линейной модели.
  2. Определите наклон и пересечение y каждой линейной модели.
  3. Найдите решение, установив две линейные функции равными другой и решив для x , или найдите точку пересечения на графике.

А теперь давайте попробуем применить эти ключевые факторы на практике. В следующем примере мы определяем, сколько разных типов билетов продано, учитывая информацию об общей выручке и количестве билетов, проданных на мероприятие.

Пример: запись и решение системы уравнений с двумя переменными

Стоимость билета в цирк составляет 25 долларов для детей и 50 долларов для взрослых. В определенный день посещаемость цирка составляет 2000 человек, а общий доход от ворот составляет 70 000 долларов.Сколько детей и сколько взрослых купили билеты?

Показать решение

Пусть c = количество детей и a = количество взрослых, посещающих школу.

Общее количество человек — 2000 человек. Мы можем использовать это, чтобы написать уравнение количества людей в цирке в тот день.

[латекс] c + a = 2 {,} 000 [/ латекс]

Доход от всех детей можно найти, умножив 25 долларов США на количество детей, [латекс] 25c [/ латекс]. Доход от всех взрослых можно найти, умножив 50 долларов.00 по количеству взрослых, [латекс] 50а [/ латекс]. Общий доход составляет 70 000 долларов. Мы можем использовать это, чтобы написать уравнение дохода.

[латекс] 25c + 50a = 70 {,} 000 [/ латекс]

Теперь у нас есть система линейных уравнений с двумя переменными.

[латекс] \ begin {собрано} c + a = 2,000 \\ 25c + 50a = 70 {,} 000 \ end {собрано} [/ latex]

В первом уравнении коэффициент при обеих переменных равен 1. Мы можем быстро решить первое уравнение для [латекса] c [/ латекса] или [латекса] a [/ латекса].Решим за [латекс] [/ latex].

[латекс] \ begin {собрано} c + a = 2 {,} 000 \\ a = 2 {,} 000-c \ end {собрано} [/ latex]

Подставьте выражение [latex] 2 {,} 000-c [/ latex] во второе уравнение для [latex] a [/ latex] и решите относительно [latex] c [/ latex].

[латекс] \ begin {align} 25c + 50 \ left (2 {,} 000-c \ right) & = 70 {,} 000 \\ 25c + 100 {,} 000 — 50c & = 70 {,} 000 \ \ -25c & = — 30 {,} 000 \\ c & = 1 {,} 200 \ end {align} [/ latex]

Подставьте [latex] c = 1 {,} 200 [/ latex] в первое уравнение для решения относительно [latex] a [/ latex].

[латекс] \ begin {align} 1 {,} 200 + a & = 2 {,} 000 \\ a & = 800 \ end {align} [/ latex]

Мы обнаружили, что 1200 детей и 800 взрослых купили в тот день билеты в цирк.

Попробуй

Билеты в цирк стоят 4 доллара для детей и 12 долларов для взрослых. Если было куплено 1650 билетов на питание на общую сумму 14 200 долларов, сколько детей и сколько взрослых купили билеты на питание?

Иногда система уравнений может помочь в принятии решения. В следующем примере мы помогаем ответить на вопрос: «Какая компания по аренде грузовиков предоставит наилучшую стоимость?»

Пример: построение системы линейных моделей для выбора компании по аренде грузовиков

Джамал выбирает между двумя компаниями по аренде грузовиков.Первый, Keep on Trucking, Inc., взимает предоплату в размере 20 долларов, затем 59 центов за милю. Второй, Move It Your Way, требует предоплаты в размере 16 долларов США, затем 63 цента за милю. Когда компания Keep on Trucking, Inc. станет лучшим выбором для компании Jamal?

Показать решение

Двумя важными величинами в этой задаче являются стоимость и количество пройденных миль. Поскольку нам нужно рассмотреть две компании, мы определим две функции.

Ввод d , пройденное расстояние в милях
Выходы K ( d ): стоимость в долларах для аренды у Keep on Trucking M ( d ) стоимость в долларах для аренды у Move It Your Way
Начальное значение Авансовый платеж: K (0) = 20 и M (0) = 16
Скорость изменения K ( d ) = 0 руб.59 за милю и P ( d ) = 0,63 доллара за милю

Линейная функция имеет вид [латекс] f \ left (x \ right) = mx + b [/ latex]. Используя скорости изменения и начальные расходы, мы можем записать уравнения

[латекс] \ begin {align} K \ left (d \ right) = 0,59d + 20 \\ M \ left (d \ right) = 0,63d + 16 \ end {align} [/ latex]

Используя эти уравнения, мы можем определить, когда Keep on Trucking, Inc. будет лучшим выбором. Поскольку все, что нам нужно сделать, это затраты, мы ищем, когда Move It Your Way будет стоить меньше, или когда [латекс] K \ left (d \ right)

Эти графики схематично показаны выше, K ( d ) выделены синим цветом.

Чтобы найти пересечение, мы приравниваем уравнения и решаем:

[латекс] \ begin {align} K \ left (d \ right) & = M \ left (d \ right) \\ 0,59d + 20 & = 0,63d + 16 \\ 4 & = 0,04d \\ 100 & = d \ \ d & = 100 \ end {align} [/ latex]

Это говорит нам о том, что стоимость проезда для двух компаний будет одинаковой, если проехать 100 миль.Либо посмотрев на график, либо отметив, что [латекс] K \ left (d \ right) [/ latex] растет медленнее, мы можем сделать вывод, что Keep on Trucking, Inc. будет дешевле, когда больше, чем Проехано 100 миль, то есть [латекс] d> 100 [/ латекс].

Приложения для систем кажутся почти бесконечными, но мы просто покажем еще одно. В следующем примере мы определяем количество 80% раствора метана, которое нужно добавить к 50% раствору, чтобы получить окончательный раствор 60%.

Пример: решение проблемы химической смеси

У химика есть 70 мл 50% раствора метана.Сколько 80% раствора она должна добавить, чтобы окончательный раствор состоял из 60% метана?

Показать решение

Мы воспользуемся следующей таблицей, чтобы помочь нам решить эту проблему со смесью:

Сумма Часть Всего
Начало
Добавить
Финал

Мы начинаем с 70 мл раствора, и неизвестное количество может быть x .Часть представляет собой проценты или концентрацию раствора 0,5 для начала, 0,8 для доп.

Сумма Часть Всего
Начало 70 мл 0,5
Добавить [латекс] х [/ латекс] 0,8
Финал [латекс] 70 + x [/ латекс] 0,6

Добавьте столбец суммы, чтобы получить окончательную сумму.Часть этого количества составляет 0,6, потому что мы хотим, чтобы окончательный раствор содержал 60% метана.

Сумма Часть Всего
Начало 70 мл 0,5 35
Добавить [латекс] х [/ латекс] 0,8 [латекс] 0,8x [/ латекс]
Финал [латекс] 70 + x [/ латекс] 0,6 [латекс] 42 + 0,6x [/ латекс]

Умножьте сумму на часть, чтобы получить сумму.обязательно распределить по последнему ряду: [латекс] (70 + х) 0,6 [/ латекс].

Если мы сложим начало и добавим записи в столбце «Итого», мы получим окончательное уравнение, которое представляет общую сумму и ее концентрацию.

[латекс] \ begin {align} 35 + 0,8x & = 42 + 0,6x \\ 0,2x & = 7 \\ \ frac {0,2} {0,2} x & = \ frac {7} {0,2} \\ x & = 35 \ конец {align} [/ latex]

35 мл 80% раствора необходимо добавить к 70 мл 50% раствора, чтобы получить 60% раствор метана.

Тот же процесс можно использовать, если к начальной и конечной сумме привязана цена, а не процент.

Ключевые концепции

  • Система линейных уравнений состоит из двух или более уравнений, состоящих из двух или более переменных, так что все уравнения в системе рассматриваются одновременно.
  • Решением системы линейных уравнений с двумя переменными является любая упорядоченная пара, которая удовлетворяет каждому уравнению независимо.
  • Системы уравнений классифицируются как независимые с одним решением, зависимые с бесконечным числом решений или несовместимые с отсутствием решения.
  • Один из методов решения системы линейных уравнений с двумя переменными — построение графиков. В этом методе мы строим уравнения на одном и том же наборе осей.
  • Другой метод решения системы линейных уравнений — подстановка. В этом методе мы решаем одну переменную в одном уравнении и подставляем результат во второе уравнение.
  • Третий метод решения системы линейных уравнений — сложение, в котором мы можем исключить переменную, добавив противоположные коэффициенты соответствующих переменных.
  • Часто необходимо умножить одно или оба уравнения на константу, чтобы упростить исключение переменной при сложении двух уравнений.
  • Любой метод решения системы уравнений приводит к ложному утверждению для несовместимых систем, потому что они состоят из параллельных линий, которые никогда не пересекаются.
  • Решение системы зависимых уравнений всегда будет верным, потому что оба уравнения описывают одну и ту же линию.
  • Системы уравнений могут использоваться для решения реальных проблем, которые включают более одной переменной, например, относящиеся к выручке, затратам и прибыли.

Глоссарий

метод сложения алгебраический метод, используемый для решения систем линейных уравнений, в которых уравнения складываются таким образом, чтобы исключить одну переменную, позволяя решить результирующее уравнение для оставшейся переменной; затем используется подстановка для решения первой переменной

точка безубыточности точка, в которой функция затрат пересекает функцию дохода; где прибыль равна нулю

согласованная система система, для которой существует единое решение для всех уравнений в системе, и это независимая система, или если существует бесконечное количество решений, и это зависимая система

функция затрат функция, используемая для расчета затрат на ведение бизнеса; обычно состоит из двух частей: постоянных затрат и переменных затрат

зависимая система система линейных уравнений, в которой два уравнения представляют одну и ту же линию; существует бесконечное количество решений зависимой системы

несовместимая система система линейных уравнений без общего решения, потому что они представляют собой параллельные линии, которые не имеют общих точек или прямых

независимая система система линейных уравнений с ровно одной парой решений [латекс] \ left (x, y \ right) [/ latex]

функция прибыли функция прибыли записывается как [латекс] P \ left (x \ right) = R \ left (x \ right) -C \ left (x \ right) [/ latex], выручка минус затраты

функция дохода функция, которая используется для расчета дохода, записывается как [латекс] R = xp [/ latex], где [latex] x = [/ latex] количество, а [latex] p = [/ latex] цена.

метод подстановки алгебраический метод, используемый для решения систем линейных уравнений, в которых одно из двух уравнений решается для одной переменной, а затем подставляется во второе уравнение для решения для второй переменной

система линейных уравнений набор из двух или более уравнений с двумя или более переменными, которые должны рассматриваться одновременно.


Система двух линейных уравнений с двумя переменными Калькулятор

[1] 2021.01.28 10:36 Мужчина / До 20 лет / Начальная школа / Ученик неполной средней школы / Очень /

Цель использования
Учеба Руководство
Комментарий / запрос
Очень полезно для быстрых ответов на 2 уравнения.

[2] 2021.01.20 20:31 Женский / 20-летний уровень / Старшая школа / Университет / аспирант / Полезно /

Цель использования
, чтобы узнать, как его использовать.

[3] 2020/12/01 19:17 Мужчина / 60 лет и старше / Инженер / Полезно /

Цель использования
Для проекта строительства моста
Комментарий / Запрос
полезно для инженеры

[4] 2020/07/23 14:40 Мужчина / До 20 лет / Старшая школа / Университет / аспирант / Очень /

Цель использования
Решающая статистика
Комментарий / запрос
Довольно хорошо

[5] 2020/06/23 12:09 Женский / Моложе 20 лет / Начальная школа / Младший школьник / Немного /

Комментарий / Запрос
Невозможно вычислить с корневыми значениями

[6] 2020/03/21 05:46 Женский / До 20 лет / Начальная школа / Младший школьник / Полезно /

Цель использования
Математическое представление / застрял на двух линейных уравнениях

[7] 2019/11/23 21:00 Мужчины / До 20 лет ars old / Высшая школа / Вуз / Аспирант / Очень /

Цель использования
Не терять время.

Добавить комментарий

Ваш адрес email не будет опубликован. Обязательные поля помечены *